Download as pdf or txt
Download as pdf or txt
You are on page 1of 72

CHEMICAL

CHAPTER
4
KINETICS
Recap of Early Classes
Chemical Kinetics helps us to understand how chemical reactions occur. In this chapter we
will study the rate of chemical reaction, mechanism of the reaction and factors controlling the rate
of reaction.

1.0 INTRODUCTION 12.0 EFFECT OF TEMPERATURE ON RATE OF


2.0 RATE/VELOCITY OF CHEMICAL REACTION
REACTION 13.0 ARRHENIUS THEORY OF REACTION
3.0 TYPES OF RATES OF CHEMICAL RATE
REACTION 14.0 CATALYST AND CATALYSIS
3.1 Average rate 14.1 General characteristics of catalyst
14.2 Comparison of rates of reaction in presence
3.2 Instantaneous rate
and absence of catalyst
3.3 Relation between reaction rates of different 15.0 MECHANISM OF A REACTION
species involved in a reaction 15.1 Elementary Reaction
4.0 FACTORS AFFECTING RATE OF 15.2 Complex Reaction
CHEMICAL REACTION
16.0 CALCULATION OF RATE LAW/ ORDER
5.0 RATE LAW (DEPENDENCE OF RATE ON
16.1 Mechanism in Which R.D.S. given
CONCENTRATION OF REACTANTS)
16.2 Mechanisms in which RDS not Specified
6.0 ORDER OF REACTION
17.0 PARALLEL REACTIONS
7.0 MOLECULARITY AND ORDER 18.0 RADIOACTIVITY
8.0 INTEGRATED RATE LAWS 18.1 Carbon Dating
8.1 Zero order reactions 18.2 Rock Dating
8.2 First order reactions 18.3 Stability of Nuclei with Respect to
8.3 Pseudo first order reaction Neutron - Proton Ratio
9.0 GRAPHICAL COMPARISON OF 18.4 Nuclear Fission
18.5 Nuclear Fusion
DIFFERENT ORDERS
EXERCISE-1
10.0 METHODS TO DETERMINE ORDER OF
EXERCISE-2
A REACTION
EXERCISE-3
10.1 Initial rate method EXERCISE-4(A)
10.2 Integrated rate law method EXERCISE-4(B)
10.3 Method of half lives EXERCISE-5
11.0 METHODS TO MONITOR THE
PROGRESS OF THE REACTION
11.1 Pressure measurement
11.2 Volume measurement
11.3 Optical rotation measurement
dfdfgfgf
Chemical Kinetics

1.0 INTRODUCTION
In the thermodynamics, we have studied whether a reaction will take place or not and if it does then upto what
extent. In this chapter (chemical kinetics) we will study about how fast a chemical reaction takes place and what
are the different factors affecting this rate of chemical reaction. How to optimise the conditions as to maximise
the output in optimum time. The last part of chapter will be dealing with the mechanism of a chemical reaction
and catalysis.

2.0 RATE/VELOCITY OF CHEMICAL REACTION


SL AL

The rate of change of concentration with time of different chemical species taking part in a chemical reaction is
known as rate of reaction of that species.
c mol / lit.
Rate = = = mol lit–1 time–1 = mol dm–3 time–1
t sec
Rate is always defined in such a manner so that it is always a positive quantity.

3.0 TYPES OF RATES OF CHEMICAL REACTION


SL AL

For a reaction R P
3.1 Average rate
SL AL

Total change in concentration c [R] [P]


= =– =
Total time taken t t t
3.2 Instantaneous rate
SL AL

Rate of reaction at a particular instant.


c dc d[R] d[P]
Rinstantaneous = tlim0 = =– =
t dt dt dt
(Negative sign for reactant because concentration decrease)
(Positive sign for product because concentration increase)
Instantaneous rate can be determined by drawing a tangent at time t on curve drawn for concentration versus
time.
Initial Rate : Instantaneous rate at ‘t = 0’ is called initial rate [slope of tangent at t = 0].

[R]0

– [R] – (c2– c1)


Reactant Conc.

c1 rav = =
t (t2– t1)
JPR\COMP.251\D\Allen(IIT-JEE Wing)\2020–21\Nurture\Che\Unit-08\Chemical Kinetics

c2 –d[R]
rinst = = – slope
dt

t1 t2 t time

3.3 Relation between reaction rates of different species involved in a reaction


SL AL

For the reaction : N2 + 3H2 2NH3


d [N2 ]
Rate of decomposition of N 2 =
dt

163
JEE-Chemistr y

d[H 2 ]
Rate of decomposition of H2 = -
dt

d[NH 3 ]
Rate of formation of NH3 =
dt
These rates are not all equal. Therefore by convention the rate of a reaction is defined as
d[N 2 ] 1 d[H 2 ] 1 d[NH 3 ]
Rate of reaction = - = - =
dt 3 dt 2 dt
Note : Rate of reaction value is dependent on the stoichiometric coefficients used in the reaction while rate of
any species will be a fixed value under given conditions.

Illustration 1. From the concentrations of R at different times given below, calculate the average rate of the reaction:
R®P during different intervals of time.
t/s 0 5 10 20 30
103 × [R]/mol L–1 160 80 40 10 2.5
Solution. We can determine the difference in concentration over different intervals of time and thus determine
the rate by dividing D[R] by Dt.

[R]1 ´ 10 3 [R]2 ´ 103 t2 t1 rav ´ 103 -[R 2 - R1 ] ´ 10 3


=
mol L-1 mol L-1 s s mol L-1s -1 [t 2 - t1 ]
160 80 5 0 16
80 40 10 5 8
40 10 20 10 3
10 2.5 30 20 0.75

Illustration 2. For each reaction below, express the rates of change of [product] and [reactant] in the correct
relationship to each other.
(a) 2O3(g) ® 3O2(g) (b) 2HOF(g) ® 2HF(g) + O2(g)
1 d[O 3 ] 1 d[O 2 ] 1 d[HOF] 1 d[HF] d[O 2 ]
Solution. (a) – =+ (b) – =+ =+
2 dt 3 dt 2 dt 2 dt dt

-d[O 3 ] 2 d -d[HOF] d[HF] 2d[O 2 ]


= [O2] =+ =+
dt 3 dt dt dt dt

Illustration 3. In a catalytic experiment involving the Haber's process, N2 + 3H2 ® 2NH3, the rate of
reaction was measured as rate = = 2 x 10-4 M.s-1. If there were no side reactions, express
the rate of reaction in terms of (a) N2 (b) H2?
d[N2 ] -1 d[H2 ] 1 d[NH3 ]
Solution. Rate of Reaction = – = =+
dt 3 dt 2 dt
JPR\COMP.251\D\Allen(IIT-JEE Wing)\2020–21\Nurture\Che\Unit-08\Chemical Kinetics

d[N2 ] d[H2 ]
(a) 2 × 10–4 = – (b) 2 × 10-4 × 3 = – = 6 × 10-4 MS-1.
dt dt

4.0 FACTORS AFFECTING RATE OF CHEMICAL REACTION


SL AL

(a) Concentration (b) Temperature (c) Nature of reactants and products


(d) Catalyst (e) pH of the solution (f) Dielectric constant of the medium.
(g) Radiations/light (h) Pressure (i) Electrical and magnetic field.
The first four factors generally affect rate of almost all reactions while other factors are specific to some reactions
only.

164
Chemical Kinetics
(a) Effect of concentration : We know from law of mass
action that Rate is proportional to concentration of reactants.
“ So rate of reaction decreases with passage of time, since
concentration of reactants decreases.
(b) Effect of temperature : Will be discussed later.

(c) Effect of nature of reactants and Products :


(i) Physical state of reactants : Gaseous state > Liquid state > Solid state
Decreasing order of rate of reaction because collisions in homogeneous system are more effective
than heterogenous system.
(ii) Physical size of reactants : As we decreases the particle size rate of reaction increases since
surface area increases.
(iii) Chemical nature of reactants : If more bonds are to be broken, the rate of reaction will be
slow. Similarly bond strength is more, rate of reaction will be slow.
(d) Effect of Catalyst :
(i) Presence of positive catalyst lower down the activation energy hence increases the rate of reaction.
(ii) presence of negative catalyst increases activation energy hence decreases the rate of reaction.
(e) Effect of pH of solution :
3+
(Tl )
Eg. Fe(CN)64– ¾¾¾ ¾® [Fe(CN)6]3–
This reaction takes place with appreciable rate in acidic medium, but does not take place in basic
medium.

(f) Effect of dielectric constant of the medium : More is the dielectric constant of the medium greater
will be the rate of ionic reactions.

(g) Effect of radiations/light : Radiation are useful for photochemical reaction.

(h) Effect of pressure : Pressure is important factor for gaseous reaction.

(i) Effect of electrical & Magnetic field : Electric and magnetic fields are rate determining factors if a
reaction involves polar species.

5.0 RATE LAW (DEPENDENCE OF RATE ON CONCENTRATION OF


REACTANTS)
SL AL

The representation of rate of reaction in terms of the concentration of the reactants is called the rate law.
It can only be established by experiments.
Generally rate law expressions are not simple and these may differ for the same reaction on conditions under
which the reaction is being carried out.
But for large number of reactions starting with pure reactants we can obtain simple rate laws. For these reactions:
Rate µ (conc.)order
Rate = K (conc.)order This is the differential rate equation or rate expression.
Where K = Rate constant = specific reaction rate = rate of reaction when concentration is unity
unit of K = (conc)1– order time–1
JPR\COMP.251\D\Allen(IIT-JEE Wing)\2020–21\Nurture\Che\Unit-08\Chemical Kinetics

Note : Value of K is a constant for a given reaction, depends only on temperature.

6.0 ORDER OF REACTION


SL AL

Let there be a reaction m1A + m2B ¾® products.


Now, if on the basis of experiment, we find that
R µ [A]P [B]q Where p may or may not be equal to m1 and similarly q may or may not be equal to m2.
p is order of reaction with respect to reactant A and q is order of reaction with respect to reactant B and
(p + q) is overall order of the reaction.

Note : Order of a reaction can be ‘zero’ or any whole number, it can be a fractional number and it can even be
negative with respect to a particular reactant. But overall order is not found to be negative for any reaction till
observed.
165
JEE-Chemistr y
Examples showing different values of order of reactions :
Reaction Rate law Order

(i) 2N2O5 (g) 4NO2 (g) + O2 (g) R = k [N2O5]1 1


(ii) 5Br– (aq) + BrO3– (aq) + 6H+ (aq) R = k [Br–] [BrO3–] [H+]2 1 + 1 + 2 = 4
3Br2 ( )+ 3H2O ( )
(iii) H2 (Para) H2 (ortho) R = k [H2 (Para) ]3/2 3/2
(iv) NO2 (g) + CO (g) NO (g) + CO2 (g) R = k [NO2]2 [CO]º 2+0=2
(v) 2O3 (g) 3O2 (g) R = k [O3]2 [O2]–1 2–1=1
(vi) H2 + Cl2 h 2 HCl R = k [H2]0[Cl2]0 0+0=0

The reaction (ii) does not take place in one single step. It is almost impossible for all the 12 molecules of the
reactants to be in a state of encounter simultaneously. Such a reaction is called complex reaction and takes
places in a sequence of a number of elementary reactions. For an elementary reaction the sum of stoichiometric
coefficients = order of the reactions. But for complex reactions order is to be experimentally calculated.

7.0 MOLECULARITY AND ORDER


SL AL

The number of molecules that react in an elementary step is the molecularity of the elementary reaction.
Molecularity is defined only for the elementary reactions and not for complex reactions. No elementary reactions
involving more than three molecules are known, because of very low probability of near-simultaneous collision of
more than three molecules.
The rate law for the elementary reaction
aA + bB products rate = k[A]a[B]b, where a + b = 1, 2 or 3.
For an elementary reaction, the orders in the rate law equal the coefficients of the reactants.
While, the order is defined for complex as well as elementary reactions and is always experimentally calculated
by the mechanism of the reaction, usually by the slowest step of the mechanism known as rate determining
step of the reaction.

Comparison B/W Molecularity and order of reaction

Molecularity of Reaction Order of Reaction

k
JPR\COMP.251\D\Allen(IIT-JEE Wing)\2020–21\Nurture\Che\Unit-08\Chemical Kinetics

2. It is always a whole number 2. It may be zero , fractional or integer.


It can neither be zero nor fractional.

3. It is derived from RDS in the mechanism of reaction. 3. It is derived from rate expression.

4. It is theoretical value. 4. It is experimental value.

5. Molecularity is in independent of 5.Order of reaction depends upon pressure


Pressure and temperature. and temperature.

166
Chemical Kinetics

Illustration 4. In the first step of the Ostwald process for synthesis of nitric acid, ammonia is converted into nitric
oxide by the high-temperature reaction :
4NH3 (g) + 5O2(g) 4NO(g) + 6H2O(g)
(a) How is the rate of consumption of O2 related to the rate of consumption of NH3 ?
(b) How are the rates of formation of NO and H2O related to the rate of consumption of NH3 ?
d[O 2 ] 5 d[NH 3 ] d[NO] d[NH 3 ] d[H 2O] 3 d[NH 3 ]
Solution (a) – =– (b) =– and =–
dt 4 dt dt dt dt 2 dt
4NH3 (g) + 5O2 (g) 4NO (g) + 6H2O (g)

1 d[NH 3 ] 1 d[O 2 ] 1 d[NO] 1 d[H 2O]


(a) Rate = – =– =+ = +
4 dt 5 dt 4 dt 6 dt

d[O 2 ] 5 d[NH 3 ]
– =–
dt 4 dt

1 d[NH 3 ] 1 d[O 2 ] 1 d[NO] 1 d[H 2O]


(b) Rate = – =– =+ = +
4 dt 5 dt 4 dt 6 dt

d[NO] d[NH 3 ] d[H 2O] 3 d[NH 3 ]


=– and =–
dt dt dt 2 dt

Illustration 5. In the following reaction


2H2O2 (aq) 2H2O ( ) + O2 (g) rate of formation of O2 is 36 g min–1 ,
(a) What is rate of formation of H2O. (b) What is rate of disappearance of H2O2.
Solution (a) 40.5 gm min–1 (b) 76.5 gm min–1
2H2O2 2H2O + O2

d[O 2 ]
= 36 gram/ min = 1.125 mole/min
dt

1 d[H 2O 2 ] 1 d[H 2O] d[O 2 ]


– = =
2 dt 2 dt dt

d[H 2O] d[O 2 ]


(a) = 2 = 2 × 1.125 Mole/min = 40.5 gram/min
dt dt

1 d[H 2O 2 ] d[O 2 ] d[H 2O 2 ] d[O 2 ]


(b) – = – = 2
2 dt dt dt dt
= 2 × 1.125 mole/min = 2 × 1.125 × 34 gram/min = 76.5 gram/min
JPR\COMP.251\D\Allen(IIT-JEE Wing)\2020–21\Nurture\Che\Unit-08\Chemical Kinetics

Illustration 6. For the reactions of I, II and III order, k1 = k2 = k3 when concentrations are expressed in mol litre 1.
What will be the relation in k'1, k'2, k'3, if concentrations are expressed in mol/mL?
Solution k'1 = k'2 × 10–3 = k'3 × 10–6
r = k [A]n k = r. [A]–n Assume r = x mole–1 sec–1
k1 = sec –1
k'1 = sec–1
k2 = mole–1 Lit sec–1 k'2 = 103 x mole–1 ml sec–1
k3 = mole lit sec
–2 2 –1
k'3 = 106 x mole–2 ml–2 sec–1
Value of k'1 = [Value of k'2] × 10 = [Value of k'3] × 10–6
–3

167
JEE-Chemistr y
Illustration 7. The following data are for the reaction A + B products:
conc. A conc. B Initial Rate
(M) (M) (mol L 1 s 1)
0.1 0.1 4.0 x 10 4
0.2 0.2 1.6 x 10 3
0.5 0.1 2.0 x 10 3
0.5 0.5 1.0 x 10 2
(i) What is the order with respect to A and B for the reaction?
(ii) Calculate the rate constant.
(iii) Determine the reaction rate when the concentrations of A and B are 0.20 M and 0.35 M,
respectively.
Ans. (i) rate = [A] [B] (ii) k = 4 x 10 2M 1s 1 (iii) rate = 2.8 x 10 3M·s 1
x x
r1 a1 4 10 –4 0.1
Solution. (i) 0 = K [A]x [B]Y r3 = =
a3 20 10 –4 0.5

x Y Y
1 1 r3 b3 2 10–3 0.1
= = x=1 r4 = =
5 5 b4 10 10 –3 0.3

Y
1 1
= = y=1
5 5
(i) Rate = K [A] [B]
(ii) 4 × 10–4 = K [0.1] [0.1] k = 4 × 10–2 n–1 sec–1
(iii) r = 4 × 10–2 (0.2) (0.35) = 2.8 × 10–3

Based on Rate of Reaction


1. The rate constant of a reaction is equal to rate of reaction-
(A) When concentrations of reactants do not change with time
(B) When concentrations of all reactants and products are equal
(C) At time t = 0
(D) When concentrations of all reactants are unity

2. For the reaction N2 + 3H2 2NH3 the rate of change of concentration for hydrogen is – 0.3 × 10 –4 Ms–1.
The rate of change of concentration of ammonia is :
(A) – 0.2 × 10–4 (B) 0.2 × 10–4 (C) 0.1 × 10–4 (D) 0.3 × 10–4

3. If the concentration of the reactants in the elementary reaction 2A + B C + D is increased by three folds, the
rate of the reaction will be increased by-
(A) 27 times (B) 9 times (C) 64 times (D) 01 times
JPR\COMP.251\D\Allen(IIT-JEE Wing)\2020–21\Nurture\Che\Unit-08\Chemical Kinetics

4. The rate of change in concentration of C in the reaction 2A + B 2C + 3D was reported as 1.0 mol litre–1
sec . Calculate the reaction rate :
–1

(A) 0.05 mole litre–1 sec–1 (B) 0.01 mol litre–1 sec–1
(C) 0.5 mol litre–1 sec–1 (D) None of these

5. Which of the following statement is not correct for the reaction- 4A + B 2C + 2D


(A) The rate of disappearance of B is one forth rate of disappearance of A
(B) The rate of appearance of C is one half the rate of disappearance of B
(C) The rate of formation of D is one half the rate of consumption of A
(D) The rates of formation of C and D are equal.

168
Chemical Kinetics

6. Substance A reacts according to a first order rate law with


k = 5.0 × 10–5 s–1. If the initial conc. of A is 1.0 M, the initial rate is
(A) 1 × 10–5 Ms–1 (B) 5.0 × 10–5 Ms–1 (C) 1 × 10–4 Ms–1 (D) 5.0 × 10–4 Ms–1

7. For the reaction, 2A + B ® Products, the following initial rates were obtained at various initial concentrations
[A] [B] Rate (mol L–1 sec–1)
0.1 M 0.2 M 0.46
0.2 M 0.2 M 1.84
0.2 M 0.1 M 0.92
The rate law for the reaction is :
(A) Rate = k [A]2 [B]0 (B) Rate = k [A] [B] (C) Rate = k [A]2 [B] (D) Rate = k [A] [B]2

8. The following kinetic data are provided for a reaction between A and B :
Concentration of A(M) Concentration of B(M) Rate of reaction (M min–1)
0.50 0.02 1.15 × 10–4
0.50 0.04 2.30 × 10–4
0.01 1.00 2.30 × 10–6
0.02 1.00 0.92 × 10–5
Then value of the rate constant for the above reaction is equal to :
(A) 1.15 × 10–4 dm3/mol min (B) 2.30 × 10–4 dm6/mol2 min
–2 6
(C) 2.30 × 10 dm /mol min 2 (D) 1.15 × 10–2 dm3/mol min

9. The following data pertain to reaction between A and B ;


S.No. [A] mol. L–1 [B] mol. L–1 Rate mol. L–1. t–1
I 1 × 10 –2
2 × 10 –2
2 × 10–4
II 2 × 10 –2
2 × 10 –2
4 × 10–4
III 2 × 10 –2
4 × 10 –2
8 × 10–4
Which of the following inference(s) can be drawn from the above data
[a] Rate constant of the reaction is 10–4
[b] Rate law of the reaction is k [A] [B]
[c] Rate of reaction increases four times on doubling the concentration of both the reactants
Select the correct answer
Codes :
(A) a, b and c (B) a and b (C) b and c (D) c alone

10. Following data are obtained for the reaction A + B ® product


[A] (mole L–1) [B] (mole L–1) Rate (mole L–1 s–1)
4 × 10 –2
4 × 10 –2
2 × 10–2
4 × 10 –2
8 × 10 –2
4 × 10–2
2 × 10 –2
8 × 10 –2
2 × 10–2
The reaction confirms the rate law
d[B] d[B] d[B] d[A]
(A) – = k [A] (B) – = k [A] [B]2 (C) – = k [A] [B] (D) – = k [B]
dt dt dt dt
JPR\COMP.251\D\Allen(IIT-JEE Wing)\2020–21\Nurture\Che\Unit-08\Chemical Kinetics

8.0 INTEGRATED RATE LAWS


[R]0
8.1 Zero order reactions
SL AL
Concentration

For a zero order reaction k = – slope of the


General rate law is, Rate = k [conc.]º = constant straight line
If C0 is the initial concentration of a reactant and Ct is the R
concentration at time ‘t’ then
C0 - C t
Rate = k = or kt = C0 – Ct or Ct = C0 – kt
't' time

169
JEE-Chemistr y
Unit of K is same as that of Rate = mol lit–1 sec–1.
C0
Time for completion =
k

C0 C0 C0
t1/2 (half life period) at t1/2 , Ct = , so kt1/2 = t1/2 =
2 2 2k
t1/2 C0

Examples of zero order reactions :


Generally decomposition of gases on metal surfaces at high concentrations follow zero order kinetics.
Ni
2PH3 (g) 2 P + 3 H2 Rate = K [PH3]º
Au
2H (g) H2 + 2
Rate = K[HI]0
Pt
2NH3(g) N2 + 3H2 Rate = K[NH3]0
hv
H2 + Cl2 2 HCl Rate = R [H2]º [Cl2]º

8.2 First Order Reactions


SL AL
(i) Let a 1st order reaction is
A Products
‘a’ 0 t =0
‘a–x’ x t = ‘t’
dx
Let be the rate of reaction at time ‘t’
dt
dx dx
= k (a–x)1 or = kdt.
dt a x
2.303 a 2.303 C0
On solving t = log or k= log C
k a x t t

Interval formula :
2.303 C 2.303 C
k= log 0 Ct = C0 e
–kt
k= log 1
t Ct (t2–t1) C2

Half life time (t1/2)


as t = t½ & Ct = C0 /2
2.303 2C 0 2.303log 2 ln2 0.693
k = t1/ 2 log C0 t1/2 = = =
k k k
Half life period for a 1 order reaction is a constant quantity.
st
JPR\COMP.251\D\Allen(IIT-JEE Wing)\2020–21\Nurture\Che\Unit-08\Chemical Kinetics

Graphical Representation
2.303 2.303
t= log Ct + log C0
k R

tan = 2.303 tan = 2.303


't' k
k
't'
log C0/Ct
or log a/a-x log Ct

170
Chemical Kinetics
First order growth reaction
For bacteria multiplication or virus growth use following concept
Consider a growth reaction
Time Population (or colony)
0 a
t (a + x)
dx dx
= k (a + x) or = kdt
dt (a x)
on integration
loge (a + x) = kt + C at t=0;x=0 C = loge a
a 2.303 a
kt = – loge =– log10 (a x)
(a x) t

2.303 a+ x
k= log10
t a

Generation time :
At t = generation time , x = a
0.693
t=
K
Examples of 1st order reactions :
(i) Decomposition of azoisopropane
CH 3 CH3
CH – N = N – CH (g) N2(g) + C6H14 (g)
CH3 CH3
(ii) (g)
Conversion of N–chloro acetanilide into p–chloroacetanilide
•• HCl
Cl – N – C – CH3 H — N – C – CH3
||
O O

Cl

1
(iii) H2O2 H2O + O (g)
2 2
(iv) NH4 NO2 2H2O + N2 (g)
(v) Radioactive decay
All radioactive decays are always first order kinetics.
226 222
Ra Rn + 2He4
88 86
JPR\COMP.251\D\Allen(IIT-JEE Wing)\2020–21\Nurture\Che\Unit-08\Chemical Kinetics

Illustration 8. The rate of a certain reaction depends on concentration according to the equation:
dC K 1C
.
dt 1 K 2C
What will be the order of reaction, when concentration (C) is :
(a) very-very high, (b) very-very low.

dC K 1C K1
Solution. (a) = 1 K C = 1
dt 2
K2
C

171
JEE-Chemistr y

1
if C is very-very high then being small may be neglected.
C

dC K1
= constant, i.e., zero order reaction.
dt K2
(b) If C is very-very low 1 + K2C K

dC K 1C K1
= × concentration
dt K K
i.e., I order reaction.

t 0.75
Illustration 9. Calculate t for a 1st order reaction :
0.50

2.303 C0 2.303 C0 t3 / 4 log 4 2log 2


Solution. k= log
1 t1/ 2 log C0 t1 / 2 = = =2
t3 / 4 log 2 log 2
C0
4 2
Illustration 10. At least how many half-lives should elapse for a 1st order reaction A products so that the
reaction is at least 95% completed ? (log 2 = 0.3)
(A) 4 (B) 5 (C) 6 (D) 7
Solution. (B)
t1 / 2 t1/ 2
100
t1 / 2
50
t1 / 2
25
t1/ 2 12.5 6.25 3.125
0% 50% 75% 87.5% 93.75% 96.875%

8.3 Pseudo first order reaction


SL AL
A second order (or of higher order) reactions can be converted into a first order reaction if the other reactant is
taken in large excess. Such first order reactions are known as pseudo first order reactions.
For A + B Products [Rate = K [A]1 [B]1
2.303 b(a x)
k= log
t(a b) a(b x)
Now if ‘B’ is taken in large excess b > > a.
2.303 (a x) 2.303 a
k= log k= log
bt a bt a x
‘b’ is very large can be taken as constant
2.303 a 2.303 a
kb = log k = log
t a x t a x
• k is pseudo first order rate constant
• K’ will have units of first order
• K will have units of second order.
JPR\COMP.251\D\Allen(IIT-JEE Wing)\2020–21\Nurture\Che\Unit-08\Chemical Kinetics

Examples of Pseudo 1st order reactions :


(a) Hydrolysis of canesugar :
C12H12O11 + H2O C6H12O6 + C6H12O6
sucrose excess glucose fructose
(b) Hydrolysis of esters :
H
CH3COOCH3 + H2O CH3COOH + CH3OH
(excess)

172
Chemical Kinetics

Illustration 11. Hydrolysis of methyl acetate in aqueous solution has been studied by titrating the liberated
acetic acid against sodium hydroxide. The concentration of the ester at different times is given
below :
t/min 0 30 60 90
c/M 0.8500 0.8004 0.7538 0.7096.
Show that it follows a pseudo first order reaction as the concentration of H2O remains nearly
constant (55.5 M) during the course of the reaction. What is the value of k in the equation?
rate = k [CH3COOCH3] [H2O]
Solution. For pseudo first order reaction, the reaction should be first order with respect to the ester when
[H2O] = constant. From the above data we note
ln (c 0 / c )
t c k[H 2O] = min–1
t
0 0.8500 –
30 0.8004 2.004 × 10 –3
60 0.7538 2.002 × 10 –3
90 0.7096 2.005 × 10 –3

It can be seen that k [H2O] is constant and equal to 2.004 × 10 –3 min–1 and hence it is pseudo
first order reaction. We can now determine k from
k [H2O] = 2.004 × 10–3 min–1
k [55.5 M] = 2.004 × 10–3 min–1
k = 3.614 × 10–5 M –1 min–1
It has the units of a second order reaction.

9.0 GRAPHICAL COMPARISON OF DIFFERENT ORDERS


AL

Concentration of reactant v/s time :

Rate v/s Concentration of reactant :


JPR\COMP.251\D\Allen(IIT-JEE Wing)\2020–21\Nurture\Che\Unit-08\Chemical Kinetics

t½ v/s Concentration of reactant :

173
JEE-Chemistr y

Illustration 12. Compounds A and B react with a common reagent with first order kinetics in both cases. If 99% of
A must react before 1% of B has reacted. What is the minimum ratio for their respective rate
2
constants ? (Given 2 - log 99 = 458)

Ans. 458
Solution. The fraction of A and B remaining at any time t, are
[A]t
(fA)t = [ A ]
0
[B] t
and (fB)t = [B]
0
\ ln (fA)t = – kA t
and ln (fB)t = – kB t
kA ln ( f A ) t ln (0.01)
\ kB = ln ( fB ) t = ln (0.99) = 458.

Illustration 13. Surface catalysed reactions that are inhibited by the products obey the rate equation (in some cases)
dx K (a - x )
= where a is the initial concentration of the reactant and K and b are constants.
dt 1 + bx
Integrate this equation. Derive an expression for t1/2 . x is the concentration of products at any time
t and the reaction is
A ¾¾® B.
dx (1 + bx )
Solution. (a - x ) = Kdt

[1 - b (a - x ) + ab]
dx (a - x )
= Kdt

1 + ab
ò dx (a - x) - ò dx × b = K ò dt
– (1 + ab) ln (a – x) – bx = Kt – (1 + ab) ln a
a
Kt = (1 + ab) ln (a - x ) – bx

a
at t = t1/2 x=
2

a b×a ba
K1/2 = (1 + ab) ln - = (1 + ab) ln 2 – = ln 2 + ab ln 2 – ba × 0.5
a 2 2
a-
2

l n 2 + ab ( l n 2 - 0 . 5 )
t1/2 = Ans.
JPR\COMP.251\D\Allen(IIT-JEE Wing)\2020–21\Nurture\Che\Unit-08\Chemical Kinetics

Illustration 14. A 1mL sample of a bacterial culture at 370C is taken, and diluted to 10 L. A 1mL sample of the
diluted culture is spread on a culture plate. Ten minutes later, another 1mL sample taken from the
original culture diluted and spread in the same way. The two plates are incubated for 24 hours. The
first sample exhibits 48 colonies of bacteria, the second 72 colonies. If we assume that each colony
originates with a single bacterium, what is the generation time (time required for doubling the
population).
Ans. 17min. (approx.)

1 æ a + x1 ö
Solution. t1 = ln ç ÷
k è a ø

174
Chemical Kinetics

1 æ a + x2 ö
t2 = ln ç ÷
k è a ø

1 æç a + x 2 ö÷
(t2 – t1) = ln ç ÷
k è a + x1 ø

1 æ a + x2 ö
k = t – t ln çç a + x ÷÷
2 1 è 1ø

1 æ 72 ö
k= ln ç ÷
10 è 48 ø

1 æ3ö
k= ln ç ÷
10 è 2 ø

æ ln 3 – ln 2 ö
k= ç 10
÷
è ø

ln 2 æ ln 3 – ln 2 ö
ç ÷
( t1/ 2 ) = è 10 ø

0.80 0.48 – 0.30


( t1/ 2 ) = 10

0.30 0.18
( t1/ 2 ) = 10

30 ´ 10
t1/2 = = 17 minutes (Approx.)
18

Based on Integrated Rate law


1. If a graph is plotted between log (a – x) and t, the slope of the straight line is equal to – 0.03. The specific
reaction rate will be :
(A) 6.9 × 10–2 (B) 6.9 (C) 0.69 (D) 6.9 × 10–4

2. Radioactive disintegration is a reaction of :


(A) First order (B) Second order (C) Zero order (D) Third order

1
3. The half life period of a chemical reaction is given by : t 1 µ
2
a n-1
The order of the reaction is :
(A) n (B) n – 1 (C) n + 1 (D) None of these
JPR\COMP.251\D\Allen(IIT-JEE Wing)\2020–21\Nurture\Che\Unit-08\Chemical Kinetics

4. In a second order reaction 20% of a substance is dissociated in 40 minutes. The time taken by the 80% of its
dissociation is
(A) 160 minutes (B) 640 minutes (C) 200 minute (D) 320 minute

5. The half life period of a first order reaction is 100 seconds. Its rate constant is :
(A) 0.693 sec–1 (B) 6.93 × 10–3 sec–1 (C) 6.93 × 10–2 sec–1 (D) None of these

6. 75% of a first order reaction was completed in 32 minutes. When was 50% of the reaction completed.
(A) 24 minutes (B) 8 minutes (C) 16 minutes (D) 4 minutes

7. The half life period of a first order reaction is 15 minute. How much reaction will be completed in 30 min.
(A) 75% (B) 80% (C) 95% (D) 100%

175
JEE-Chemistr y
8. 99% at a first order reaction was completed in 32 min. When will 99.9% of the reaction complete.
(A) 48 min (B) 46 min (C) 50 min (D) 45 min

9. t0.5 = constant, confirms the first order of the reaction as one a 2 t0.5 = constant confirms that the reaction is of
(A) Zero order (B) First order (C) Second order (D) Third order

10.0 METHODS TO DETERMINE ORDER OF A REACTION


10.1 Initial rate method
AL

By comparison of different initial rates of a reaction by varying the concentration


of one of the reactants while others are kept constant
r = k [A]a [B]b [C]c if [B] = constant
[C] = constant
then for two different initial concentrations of A we have
r01 = k [A0]1a r02 = k [A0]2a

a
r01 [A 0 ]1
r02 [A 0 ]2

log r01 r02


or in log form we have a=
log [A 0 ]1 [A 0 ]2

10.2 Integrated rate law method


AL
It is method of hit and trial. By checking where the kinetic data (experimental data) best fits into which
integrated rate law, we determine the order. It can also be done graphically.

10.3 Method of half lives


AL
The half lives of each order is unique so by comparing half lives we can determine order
1
for nth order reactiont1/2
[R0 ]n 1

t1 / 2 1
n 1
(R 0' )2
n 1
t1/' 2 2
(R 0 )1

Illustration 15. The rate of decomposition of N2O5 in CCl4 solution has been studied at 318 K and the following
results have been obtained :
t/min 0 135 342 683 1693
c/M 2.08 1.91 1.67 1.35 0.57
JPR\COMP.251\D\Allen(IIT-JEE Wing)\2020–21\Nurture\Che\Unit-08\Chemical Kinetics

Find the order of the reaction and calculate its rate constant. What is the half-life period?
Solution. It can be shown that these data will not satisfy the integrated rate law of zero order. We now try
ln (c 0 / c )
integrated first order equation i.e., k=
t
ln (c 0 / c )
t/min c/M k= min–1
t
0 2.08 6.32 × 10–4
135 1.91 6.30 × 10–4
339 1.68 6.32 × 10–4
683 1.35 6.32 × 10–4
1680 0.72 6.31 × 10–4

176
Chemical Kinetics
It can be seen that the value of k is almost constant for all the experimental results and hence it is first order
reaction with k = 6.31 × 10–4 min–1 .
0.69
t1/2 = = 1.094 × 103 min–1
6.31 10 4 min 1
Graphical method : Alternatively, if we draw a graph between ln c against t, we obtain a straight line with slope
= – k.

Illustration 16. In the reduction of nitric gas with hydrogen, the reaction was found to be 50% complete in 210
seconds when the initial pressure of the mixture was 200 mm. In a second experiment the time of
half reaction was 140 seconds when the initial pressure was 300 mm. Calculate the total order of the
reaction.
1
Solution. For a nth order reaction (n 1), t1/2
c n0 1

n 1
210 300
= n=2
140 200

11.0 METHODS TO MONITOR THE PROGRESS OF THE REACTION


11.1 Pressure measurement
AL
Progress of gaseous reaction can be monitored by measuring total pressure at a fixed volume &
temperature.
This method can applied for those reaction also in which a gas is produced because of decomposition of a solid
or liquid. We can get an idea about the concentration of reacting species at a particular time by measuring
pressure.

The pressure data can be given in terms of


(i) Partial pressure of the reactant
(ii) Total pressure of the reaction system
(iii) Pressure at only some points of time

Illustration 17. Find the expression for K in terms of P0 , Pt and n


Solution. Let there is a 1st order reaction
A(g) nB(g)
Let initial pressure at time t = 0 P0 0
pressure at time t P0 – x nx
Pt (Total pressure at time ‘t’) = P0 – x + nx = P0 + (n – 1) x
Pt P0
x=
n 1
Pt P0 P0n Pt
PA = P0 – =
JPR\COMP.251\D\Allen(IIT-JEE Wing)\2020–21\Nurture\Che\Unit-08\Chemical Kinetics

n 1 n 1
nP0 Pt
a p0 & a–x PA =
n 1

2.303 P0 (n 1)
k= log nP P
t 0 t
Final total pressure after infinite time = Pf = nP0
Formula is not applicable when n = 1, the value of n can be fractional also.
Do not remember the formula but derive it for each question.

177
JEE-Chemistr y
Illustration 18. The following data are obtained for the hydrolysis of benzene diazonium chloride.
C6H5N2Cl(aq) + H2O( ) C6H5OH(aq) + HCl(aq) + N2(g)

Time (minutes) 0 2 8 16 30 50
Pressures (P) of N2 0 1.6 6.2 11.2 15.5 24.4 34.0
at constant vol.
(arbitrary units)
Calculate the rate constant, assuming first order kinetics.
34 34 34 34 34
[ n = 0.0482, n = 0.2013, n = 0.3996, n = 0.6086, n = 1.2646]
32 .4 27 .8 22 .8 18.5 9.6

Ans. 2.498 × 10-2 min-1


Solution. C6H5N2Cl(aq) + H2O( ) C6H5OH(aq) + HCl(aq) + N2(g)

2 .303 P – Po
k= log Here PO = 0
t P – Pt

2 .303 34 – 0 2 .303 34 2 .303


= log 34 – Pt = log 32.4 = × 0.02 = 2.4 × 10–2 min–1
t 2 2

Now we can calculate k for other combination, finally k = 2.4 × 10 –2 m–1.

11.2 Volume measurement


AL

(i) By measuring the volume of product formed we can monitor the progress of reactions.
(ii) By titration method : By measuring the volume of titrating agent we can monitor amount of reactant
remaining or amount of product formed at any time. It is the titre value . Here the milliequivalent or
millimoles are calculated using valence factors.

Illustration 19. Study of a reaction whose progress is monitored by measuring the volume of a escaping gas.
NH4NO2 (s) 2H2O ( ) + N2 (g)
Solution. Let, Vt be the volume of N2 collected at time ‘t’
V = be the volume of N2, collected at the end of the reaction.
a V and x Vt
(a – x) V – Vt
2.303 V
k= log
t V Vt

Illustration 20. From the following data show that the decomposition of hydrogen peroxide in aqueous solution is a
first - order reaction. What is the value of the rate constant?
JPR\COMP.251\D\Allen(IIT-JEE Wing)\2020–21\Nurture\Che\Unit-08\Chemical Kinetics

Time in minutes 0 10 20 30 40
Volume V in mL 25 20 15.7 12.5 9.6
where V is the number of ml of potassium permagnate required to react with a definite volume of
hydrogen peroxide solution.
Solution. The equation for a first order reaction is 2H2O2 2H2O ( ) + O2 (g)
the volume of KMnO4 used, evidently corresponds to the undecomposed hydrogen peroxide.
Hence the volume of KMnO4 used, at zero time corresponds to the initial concentration a and the
volume used after time t, corresponds to (a – x) at that time. Inserting these values in the above
equation, we get
2 .303 25
When t = 10 min. k1 = log = 0.022318 min–1 = 0.000372 s–1
10 20 .0

178
Chemical Kinetics

2 .303 25
when t = 20 min. k1 = log = 0.023265 min–1 = 0.000387 s–1
20 15.7

2 .303 25
when t = 30 min. k1 = log = 0.02311 min–1 = 0.000385 s–1
30 12.5

2 .303 25
when t = 40 min. k1 = log = 0.023932 min–1 = 0.0003983 s–1
40 9.6
The constancy of k, shows that the decomposition of H2O2 in aqueous solution is a first order
reaction. The average value of the rate constant is 0.0003879 s –1 .

Illustration 21. Conversion of N–chloro acetanilide into p–chloroacetanilide

HCl

The above reaction is first order reaction and its progress is monitored by iodometric titration in
which liberated iodine is titrated against a standard solution of Hypo using starch as indicator. Given
that in this reaction KI does not react with the product (p–chloro acetanilide).
Calculate the rate constant of the reaction. Given that volume of hypo consumed at t = 0 is V0 and
at time ‘t’, Vt
Solution. Let, V0 = volume of hypo consumed at t = 0
Similarly Vt = volume of hypo consumed at t = ‘t’
a V0 { K reacts with the reactant only}
a – x Vt
2.303 V0
k= log
t Vt

Illustration 22. Study of acid hydrolysis of an ester.


H
CH3COOCH3 + CH3COOH + CH3OH
( HCl)

The progress of this reaction is monitored or determined by titrating the reaction mixture at different
time intervals against a standard solution of NaOH using phenolphthalein as indicator. Find out rate
constant of the reaction in terms of volume of NaOH consumed at t = 0, V 0 , at t = , V & at time
t, Vt .
Solution. Let,V0 = vol. of NaOH used at t = 0 [this is exclusively for HCl.]
Vt = vol. of NaOH used at ‘t’
V = vol. of NaOH used at t =
a V – V0
a – x V – Vt
x Vt – V0
V V0
JPR\COMP.251\D\Allen(IIT-JEE Wing)\2020–21\Nurture\Che\Unit-08\Chemical Kinetics

2 .303
a V – V0 ; k= log V Vt
t

11.3 Optical rotation measurement


AL
It is used for optically active sample. It is applicable if there is at least one optically active species involved
in chemical reaction.
The optically active species may be present in reactant or product.

It is found that (r r0 ) a (a = concentration , x = amount consumed)


(r rt ) (a x )
where are r0, rt, r are angle of optical rotation at time t = 0, t = t and t =

179
JEE-Chemistr y

Illustration 23. Study of hydrolysis of sucrose progress of this reaction is monitored with the help of polarimeter
because a solution of sucrose is dextrorotatory and on hydrolysis, the mixture of glucose as fructose
obtained becomes laevorotatory. That’s why this reaction is also known as inversion of cane sugar.
H
C12H22O11 + H2O C6H12O6 + C6H12O6
excess glucose fructose
Sp. rotation +66.5º +52.7º –92.4º
Let the readings in the polarimeter are
t = 0 , 0 ; t = ‘t’, t and at t =
Then calculate rate constant ‘k’ in terms of these readings.
Solution. The principle of the experiment is that change in the rotation is directly proportional to the amount
of sugar hydrolysed.
a – 0; a–x – t
2 .303 0
k= log
t t

12.0 EFFECT OF TEMPERATURE ON RATE OF REACTION


SL AL

In early days the effect of temperature on reaction rate was expressed in terms of temperature coefficient
which was defined as the ratio of rate of reaction at two different temperature differing by 10ºC (usually these
temperatures were taken as 25ºC and 35ºC)
K t 10
T.C. = Kt 2 to 3 ( for most of the reactions)

Note. For some reactions temperature coefficient is also found to be less than unity. For example
2NO + O2 2NO2 rate of reaction decreases on increasing temperature.

k 40 º C
Illustration 24. For a reaction T.C. = 2, Calculate for this reaction.
k 25 º C
t 15 3
k2
Solution. ( T .C .) 10 (2) 10 ( 2) 2 8
k1
Note. But the method of temperature coefficient was not exact and to explain the effect of temperature
on reaction rate new theory was evolved

13.0 ARRHENIUS THEORY OF REACTION RATE


SL AL

It was developed by Max Trautz and William lewis.


JPR\COMP.251\D\Allen(IIT-JEE Wing)\2020–21\Nurture\Che\Unit-08\Chemical Kinetics

It gives insight in to the energetics and mechanistic aspects of reactions.


It is based upon kinetic theory of gases.
Arrhenius proposed a theory of reaction rate which states as follows :
(i) A chemical reaction takes place due to the collision among reactant molecules. The number of collisions
taking place per second per unit volume of the reaction mixture is known as collision frequency (Z).
(ii) Every collision does not bring a chemical change. The collision that actually produce the products are
effective collision. For a collision to be effective the following two barriers are to be cleared.
Energy barrier : The minimum amount of energy which the colliding molecules must possess as to make the
chemical reaction to occur is known as threshold energy.
“The minimum amount of energy required by reactant molecules to participate in a reaction is called activation
energy (Ea)”

180
Chemical Kinetics

Orientation barrier : Energy alone does not determine the effectiveness of the collision. The reacting molecules
must collide in proper direction to make collision effective. Following diagrams can explain importance of
suitable direction for collision.

A A
A A
A A
B B
B B B B
Reactants
Transition Product
state

(iii) Collision to be effective the colliding molecules must possess some certain minimum energy called threshold
energy of the reaction.
(iv) Reactant molecules having energy equal or greater than the threshold are called active molecules and
those having energy less than the threshold are called passive molecules.
(v) At a given temperature there exists a dynamic equilibrium between active and passive molecules. The
process of transformation from passive to active molecules being endothermic, increase of temperature
increases the number of active molecules and hence the reaction.
Passive molecules Active molecules, H = +ve
(vi) Concept of energy of activation (Ea)
• The extra amount of energy which the reactant molecules (having energy less than the threshold) must
acquire so that their mutual collision may lead to the breaking of bond(s) and hence the energy is known
as energy of activation of the reaction. It is denoted by the symbol E a. Thus,
Ea = Threshold energy – Actual average energy
Ea is expressed in kcals mole–1 or kJ mole–1.
• The essence of Arrhenius Theory of reaction rate is that there exists an energy barrier in the reaction path
JPR\COMP.251\D\Allen(IIT-JEE Wing)\2020–21\Nurture\Che\Unit-08\Chemical Kinetics

between reactant(s) and product(s) and for reaction to occur the reactant molecules must climb over the
top of the barrier which they do by collision. The existence of energy barrier and concept of Ea can be
understood from the following diagram.

HR = Summation of enthalpies of reactants


Threshold enthalpy
Enthalpy (H) Ea1 Ea2 or energy HP = Summation of enthalpies of reactants
H = Enthalpy change during the reaction
HR
Reactants Ea1 = Energy of activation of the forward reaction
H = Hp – HR = Ea1 – Ea2 Ea2 = Energy of activation of the backward reaction
HP
Products

Progress of reaction (or reaction coordinate)

181
JEE-Chemistr y
From the figure above it can be concluded that the minimum activation energy of any exothermic reaction will
be zero while minimum activation energy for any endothermic reaction will be equal to H.
Greater the height of energy barrier, greater will be the energy of activation and more slower will be the reaction
at a given temperature.

Rate of any chemical reaction = Collision frequency × fraction of the total number of effective collision
= Collision frequency × fraction of the total number of collision in which
K.E. of the colliding molecules equals to Ea or exceeds over it.
Collision frequency is the number of collisions per unit volume per unit time. It is denoted by the symbol Z. Z is
directly proportional to T . By 10ºC rise in temperature, so it is the fraction of the total number of effective
collision that increases markedly resulting into marked increase in the reaction rate.
From maxwellian distribution it is found that fraction of molecules having excess energy greater than threshold
energy lead to the formation of product.

Fraction of T2
molecule T1 T2 > T1
Ea

e Ea / RT represents fraction of molecules K.E. having energy greater Ea

rate e Ea / RT
dependence of rate on temperature is due to dependence of k on temperature.

k e E a / RT
k Ae Ea / RT (Arrhenius equation)
A is pre exponential factor / frequency factor representing collisions taking place with proper orientation.
A and Ea are independent of temperature generally.
JPR\COMP.251\D\Allen(IIT-JEE Wing)\2020–21\Nurture\Che\Unit-08\Chemical Kinetics

Ea = min K. E. colliding molecules must have to reach transition state.


Ea
nk = n A – Ea 0
RT
As T ,K A

Reversible Reactions
E af / RT
kf = Af e
E ab / RT
kb = Ab e

182
Chemical Kinetics

Eaf / RT
kf Af e Af
(E af Eab ) / RT
Keq = k = Eab / RT = Ab e
b Ab e

H Af
n Keq = – + n A
RT b

At temperature T1, rate constant = k1


At temperature T2, rate constant = k2
Ea Ea k2 E 1 1
nk1 = nA – nk2 = nA – n = a T1 T2 (remember)
RT1 RT2 k1 R

Illustration 25. Explain on the basis of temprature coeff. of rate const. that equilibrium of endothermic reaction
shifts in forward direction on increasing temprature while equilibriums of exothermic shift back.
K eq1 H 1 1
n =
K eq 2 R T1 T2

Solution. H = Eaf – Eab > 0 (for endothermic)


Eaf > Eab
on increasing temp. kf more than kb
equilibrium will shift in forward direction.

14.0 CATALYST AND CATALYSIS


AL

A catalyst is a substance, which increases the rate of a reaction without itself being consumed at the end of the
reaction, and the phenomenon is called catalysis.

Catalyst are generally foreign substances but sometimes one of the product may act as a catalyst and such
JPR\COMP.251\D\Allen(IIT-JEE Wing)\2020–21\Nurture\Che\Unit-08\Chemical Kinetics

catalyst is called “auto catalyst” and the phenomenon is called auto catalysis.
Examples of catalysis
(a) Thermal decomposition of KCIO3 is found to be accelerated by the presence of MnO2. Here MnO2 acts as
a catalysts.
2KClO3 + [MnO2] 2KCl + 3O2 + [MnO2]
MnO2 can be received in the same composition and mass at the end of the reaction.

(b) In the permanganate titration of oxalic acid initially there is slow discharge of the colour of permanganate
solution but afterwards the discharge of the colour become faster. This is due to the formation of MnSO 4
during the reaction which acts as a catalyst for the same reaction. Thus, MnSO4 is an “auto catalyst”
for this reaction. This is an example of auto catalyst.
2KMnO4 + 3H2SO4 + 5H2C2O2 K2SO4 + 8H2O + 10CO2
183
JEE-Chemistr y
14.1 General characteristics of catalyst
AL
(i) A catalyst does not initiate the reaction. It simply fastens it.
(ii) Only a small amount of catalyst can catalyse the reaction.
(iii) A catalyst does not alter the position of equilibrium i.e. magnitude of equilibrium constant and hence
Gº. It simply lowers the time needed to attain equilibrium. This means if a reversible reaction in absence
of catalyst completes to go to the extent of 75% till attainment of equilibrium, and this state of equilibrium
is attained in 20 minutes then in presence of a catalyst the reaction will go to 75% of completion before
the attainment of equilibrium but the time needed for this will be less than 20 minutes.

(iv) A catalyst drives the reaction through a low energy Ea


P.E.
path and hence Ea is less. That is, the function of the
E'a
catalyst is to lower down the activation energy.
Ea = Energy of activation in absence of catalyst. HR
E’a = Energy of activation in presence of catalyst. HP
Ea – E’a = lowering of activation energy by catalyst. Products

Reaction Coordinate

14.2 Comparison of rates of reaction in presence and absence of catalyst


AL
If k and kcat be the rate constant of a reaction at a given temperature T, and E a and E’a are the activation energies
of the reaction in absence and presence of catalyst, respectively, the
E'a / RT
k cat Ae
Ea / RT = Ae (Ea E 'a ) / RT
k Ae
k cat
Since Ea – E’a is positive so kcat > k. the ratio gives the number of times the rate of reaction will increase
k
by the use of catalyst at a given temperature
The rate of reaction in the presence of catalyst at any temperature T 1 may be made equal to the rate of reaction
in absence of catalyst but for this sake we will have to raise the temperature. Let this temperature be T 2 =
E' a Ea
e E'a / RT1
e E a / RT2 or
T1 T2

Catalyzed pathway Uncatalyzed pathway


Potential energy

Potential energy

Ea Ea

Reactants Reactants

Products Products

Reaction progress Reaction progress


(a) (b)
JPR\COMP.251\D\Allen(IIT-JEE Wing)\2020–21\Nurture\Che\Unit-08\Chemical Kinetics

Illustration 26. The pyrolysis of an organic ester follows a first order process and its rate constant can be
42075
expressed as ln k = 78.09 –
T
where k is given in the min–1. Calculate the time required for 25 percent reaction to complete at
227°C.
42075
Solution. ln k = 78.09 – = – 6.06
500

6.06
log k = – = – 2.63 ; k = 2.344 × 10–3 min –1
2.303

184
Chemical Kinetics

2.303 a 2.303
when x = 0.25 ; k = log 0.75 a t 1/4 = 3 log 1.333 = 123.06 min
t 1/ 4 2.344 10

Illustration 27. From the following data for the reaction between A and B.
–3 –1
[A] [B] Initial rate (mole dm min )
–3 –3
(mole dm ) (mole dm )
300 k 320 k
(i) 2.5 × 10
–4
3.0 × 10
–5
5.0 × 10
–4 2.0 × 10–3
–4 –5 –3
(ii) 5.0 × 10 6.0 × 10 4.0 × 10
–3 –5 –2
(iii) 1.0 × 10 6.0 × 10 1.6 × 10
Calculate the following
(i) The order w.r.t. A and w.r.t. B (ii) The rate constant at 300 K
(iii) The energy of activation (iv) The pre-exponential factor
[e –22.18 = 2.3283 × 10–10 , ln2 = 0.693]
Ans. (i) 2, 1 (ii) 2.67 × 108 mol–2 litre2 sec–1
(iii) 55.13 kJ mol–1 (iv) 1.147 × 1018 mol–2 litre2 sec–1
Solution. (i) r = K [A]x [B]Y
From (2 & 3) equation
x
5 10 – 4
x 2
4 10 –3 1 1
= = = x= 2
10 10– 4 16 10 – 3 2 2

From 1 & 2
x y x y
2.5 10 – 4 3 10 – 5 5 10–4 1 1 1
–4 –5 = –4 = = 23 y=1
5 10 6 10 40 10 2 2

(ii) Rate constant at 300 K


5 10 –4
r = K [A]x [B]Y = 5 × 10–4 K300 =
( 2.5 10 – 4 )2 (3 10 – 5 )

5 10 –4 50 108
K300 = –8 –5 = K300 = 2.67 × 108 Mole–2 Lit2 sec–1
6.25 10 3 10 6.25 3

K2 Ea 1

1 2 10 –3
(iii) log K1 = 2.303 R T1 T2 K320 =
6.25 10 – 8 3 10 – 5

200 10 8
= = 10.67 × 10+8
6.25 3

10.67 108 Ea 320 – 300 Ea 20


log = × 0.60 = 2.303 8.314
JPR\COMP.251\D\Allen(IIT-JEE Wing)\2020–21\Nurture\Che\Unit-08\Chemical Kinetics

2.67 10 8 2 .303 8 .314 320 300 320 300

Ea
K = A e– Ea = 55.13 kJ/mole
RT

55.32 103
2.67 × 108 = A e– = A e– 22.18
8.314 300
2.67 × 108 = A × 2.3283 × 10–10 ,

2.67 108
A= = 1.147 × 1018
2.3283 10–10

185
JEE-Chemistr y

Based on Factors affecting Rate of Reaction


1. The rate of a certain reaction increase by 2.3 times when the temperature is raised from 300 K to 310K. If k is
the rate constant at 300 K then the rate constant at 310 K will be
(A) k (B) 3k2 (C) 2k (D) 2.3k

2. In a reaction, the threshold energy is equal to-


(A) Activation energy (B) Activation energy - normal energy of reactants
(C) Activation energy + normal energy of reactants (D) Normal energy of reactants

3. The activation energies for forward and backward reactions are Ef and Eb respectively. If the reaction is
exothermic then :
(A) Ef > Eb (B) Ef < Eb
(C) Ef = Eb (D) There is no relation between Ef and Eb directly

1 1
4. The slope of the line for the graph of log k versus for the reaction, N2O5 2NO2 + O is – 5000. Calculate
T 2 2
the energy of activation of the reaction - (kJ K –1 mol—1)
(A) 95.7 (B) 9.57 (C) 957 (D) None

5. For a reaction, the rate constant is expressed as, k = A.e –40000/T . The energy of the activation is :
(A) 40000 cal (B) 88000 cal (C) 80000 cal (D) 8000 cal

6. A B KA = 1015 e–2000/T
C D KC = 1014 e–1000/T
Temperature TK at which (KA = KC) is :
(A) 1000 K (B) 2000 K (C) (2000 / 2.303) K (D) (1000 / 2.303) K

7. For the first order reaction A B + C, carried out at 27°C if 3.8 × 10–16 % of the reactant molecules exists
in the activated state, the Ea (activation energy) of the reaction is :
(A) 12 kJ/mol (B) 831.4 kJ/mol (C) 100 kJ/mol (D) 88.57 kJ/mol

15.0 MECHANISM OF A REACTION


AL

Reactions can be divided into


(i) Elementary / simple / single step
(ii) Complex / multi-step

15.1 Elementary Reaction


JPR\COMP.251\D\Allen(IIT-JEE Wing)\2020–21\Nurture\Che\Unit-08\Chemical Kinetics

AL

(i) These reaction take place in single step without formation of any intermediate
T.S.

P.E. Ep

Er

Reaction coordinates
(ii) For elementary reaction we can define molecularity of the reaction which is equal to no of molecules
which make transition state or activated complex because of collisions in proper orientation and with
sufficient energy
186
Chemical Kinetics

(iii) molecularity will always be a natural no


1 = unimolecular one molecule gets excited (like radioactivity)
2.= bimolecular
3 = trimolecular
(iv) Molecularly 3 because the probability of simultaneous collision between 4 or more molecules in proper
orientation is very low
(v) For elementary reaction there is only single step and hence it is going to be rate determining step so order
of an elementary reaction is its molecularity
Order of elementary reaction w.r.t. reactant = stoichiometric co-efficient of the reactant
H2 + 2
2H (Simple reaction) rate = k [H2] [ 2]
2H2 + 2 2 4H ( not elementary )
reaction obtained by multiplying an elementary reaction with some no will not be of elementary nature
H2 + Cl2 2HCl order = 0

15.2 Complex Reaction


AL

(i) Reaction which proceed in more than two steps. or having some mechanism. ( sequence of elementary
reaction in which any complex reaction procceds)
T.S.

E intermediate

Reaction coordinates

(ii) For complex reaction each step of mechanism will be having its own molecularity but molecularity of net
complex reaction will not be defined.
(iii) Order of complex reaction can be zero fractions whole no, even negative w.r.t. some species.
(iv) Order of reaction or rate law of reaction is calculated with the help of mechanism of the reaction
generally using Rate determine step (R.D.S) if given.
(v) Rate law of a reaction is always written in terms of conc. of reactant, products or catalysts but never in
terms of conc. of intermediates.
The mechanism of any complex reaction is always written in terms of elementary steps, so molecularity
of each of these steps will be defined but net molecularity of complex reaction has no meaning.
The mechanism of most of the reaction will be calculated or predicted by using mainly the following
approximations.

16.0 CALCULATION OF RATE LAW/ ORDER


16.1 Mechanism in Which R.D.S. given
AL
(i) If R.D.S. involves only reactant, product or catalyst on reactant side
rate law of R.D.S. = rate law of reaction
(ii) RDS is having intermediate on reactant side
JPR\COMP.251\D\Allen(IIT-JEE Wing)\2020–21\Nurture\Che\Unit-08\Chemical Kinetics

To calculate order, we have to specify [ intermediate] in expression of rate law in terms of conc. of [R], [P]
or catalyst with the help of same equilibrium step given in mechanism.

Illustration 28. Calculate order and rate law of reaction


2NO2 + F2 2NO2F with help of mechanism
Solution. (I) NO2 + F2 K1 NO2F + F (slow )
molecularity = 2 for both
(II) NO2 + F K2 NO2F (fast )
According to RDS
Rate = k1 [NO2] [F2]

187
JEE-Chemistr y
Illustration 29. Calculate rate law
2O3 3O2
k1
O3 O2 + O (fast eq. step)
k2
(intermediate)
O + O3 k3 2O2 (slow)
Solution. From R.D.S., rate = k3 [O3] [ O]
According to equilibrium step.
k1 [O 2 ][O]
Keq. = =
k2 [O 3 ]

k 1 [O3 ]
[O] = k [O ]
2 2

k 1 [O3 ]
Rate = k3 [O3] [O] = k3 [O3] k [O ] .
2 2

k 1 k 3 [O 3 ] 2
Rate =
k 2 [O 2 ]

16.2 Mechanisms in which RDS not Specified


AL

Steady State Approximation


Initially, for most of the cases only reactants are taken and hence the rate of production of intermediate is large
in comparison to its rate of consumption but after some time rate of consumption of intermediate will become
equal to its rate of production. This is known as steady state of reaction. Finally when reaction is going to get
completed, rate of consumption will become more then rate of production. But for most of the times reaction
remains at steady state. So rate law of reaction during steady state can be taken to be final or net rate law of
reaction.
Intermediate

Steady state

Time

d [int ermediate ]
At steady state =0
dt
JPR\COMP.251\D\Allen(IIT-JEE Wing)\2020–21\Nurture\Che\Unit-08\Chemical Kinetics

Illustration 30. 2O3 3O2


O3 k1 O2 + O

O2 + O k2 O3

O3 + O k3 2O2

1 d[O 3 ] 1 d[O 2 ]
Solution. rate = =
2 dt 3 dt

188
Chemical Kinetics

d[O 3 ]
= – k1 [O3] + k2 [O2][O] – k3 [O3] [O]
dt

d[O 2 ]
= k1 [O3] – k2 [O2] [O] + k3 [O3] [O]
dt

d [O]
At steady state =0
dt

d[O]
= k1 [O3] – k2 [O2] [O] – k3[O3] [O] = 0
dt

k1[O 3 ]
[O] = k [O ] k [O ]
2 2 3 3

d[O 3 ] k 2 [O 2 ] k1[O 3 ] k1 k 3 [O 3 ]2
= – k1 [O3] + –
dt k 2 [O 2 ] k 3 [O 3 ] k 2 [O 2 ] k 3 [O 3 ]

k1k 2 [O 2 ] [O 3 ] – k1k 3 [O 3 ] 2
= – k1 [O3] +
k 2 [O 2 ] k 3 [O 3 ]

k1k 2 [O 2 ][O3 ] – k1k 3 [O 3 ] 2 k1k 2 [O 2 ][O 3 ] – k1k 3 [O 3 ] 2


=
k 2 [O 2 ] k 3 [O 3 ]

– 2k1 k 3 [O3 ] 2
=
k 2 [O 2 ] k 3 [O 3 ]

–1 d k1 k 3 [O 3 ] 2
[O3 ] =
2 dt k 2 [O 2 ] k 3 [O 3 ]

1 d
Rate = – [O3]
2 dt

k1 k 3 [O 3 ]2
So, Rate (r) =
k 2 [O 2 ] k 3 [O 3 ]
if 3rd step is RDS then k1>> k3
k2 >> k3

k1 k 3 [O 3 ]2
r=
k 2 [O 2 ] k 3 [O 3 ]
JPR\COMP.251\D\Allen(IIT-JEE Wing)\2020–21\Nurture\Che\Unit-08\Chemical Kinetics

Illustration 31. H+ + HNO2 + C6H5NH2 Br


C6H5N2+ + 2H2O
k1
H+ + HNO2 H2NO2+
k2
H2NO2+ H+ + HNO2
k3
HNO2+ + Br– NOBr + H2O..
k4
NOBr + C6H5NH2 C6H5N2+ + H2 O + Br–

d[H2NO 2 ]
Solution. = k1 [H+][HNO2] – k2 [H2NO2+] – k3 [H2 NO2+] [Br–] = 0
dt

189
JEE-Chemistr y

[H ]]HNO 2 ]
[H2NO2+] = k1
k2 k 3 [Br – ]

d d
rate = [C 6H5NH2 ] = [HNO 2 ]
dt dt

d
[C6H5NH2 ] = k [ONBr ] [C H NH ]
dt 4 6 5 2

d
[NOBr ] = – k4 [C6H5NH2][NOBr ] + k3 [Br–] [H2NO2+] = 0
dt

k 3 [Br – ][H2NO 2 ]
[NOBr] =
k 4 [C6H5NH2 ]

k 4 k 3 [Br – ][H2NO 2 ]
r = [C 6H5NH2 ]
k 4 [C 6H5NH2 ]

k1 k 3 [Br – ][H ][HNO 2 ]


r =
k2 k 3 [Br – ]

Illustration 32. Drive a rate law for the reaction H2 (para) H2 (ortho) with the help of following mechanism.

H2 (para) 2H fast equilibrium


k3
H2 (para) + H H2 (ortho) + H slow step

k1
Ans. Rate = k3 3/2
k 2 [H2 (para)]

Solution. H2 (para) H2 (ortho)

H2 (para) 2H fast equilibrium


k3
H2 (para) + H H2 (ortho) + H slow step
1/ 2
K1 [H]2 K1
K2 = ; [H+] = H2 (para )
[H2 (para )] K2
Rate = K3[H2 (para)] [H]
1/ 2
K1
Rate = K3[H2 (para)] × H2 (para )
K2
JPR\COMP.251\D\Allen(IIT-JEE Wing)\2020–21\Nurture\Che\Unit-08\Chemical Kinetics

k1
Rate = k3 3/2
k 2 [H2 (para)]

17.0 MISCELLANEOUS REACTIONS


AL k1
(i) Parallel Reactions : These are reactions in which reaction sub- B
stances do not follow a particular path to give a particular set of
products. It follows one or more paths to give different products, A
e.g.
C
k2

190
Chemical Kinetics

The reactant A follows two different paths to form B and C as shown below :

C
A
B

Conc.

d[A]
Rate = – = k1[A] + k2[A] = k[A] [As, (k1 + k2) = k]
dt
[A]0 [B] k1
ln [A] = (k1 + k2) t ; [C] k2
t

k1[A0 ] k 2[A 0 ] k 2t
[A] = [A0] e–kt; [B] = (1 e k1t
) ; [C] = k k (1 e )
k1 k 2 1 2

k1 k2
% yield of B = × 100 ; % yield of C = k k 2 × 100
k1 k2 1

(ii) Consecutive Reaction : Frequently a product of one reaction becomes a reactant in a subsequent
reaction. This is true in multistep reaction mechanisms. We shall consider only the simple case of two
consective irreversible first-order reactions : A B with rate constant k1, and B C with rate contant k2:
k k
A 1
B 2
C
where for simplicity we have assumed stoichiometric coefficients of unity. Since the reactions were assumed
to be first-order, Thus
d [B] d [B] d [B]
= + = k1[A] – k2[B]
dt dt 1 dt 2
We have
d [A ] d [B] d [C]
= – k1[A], = k1[A] – k2 [B], = k2[B]
dt dt dt
Let only A be present in the system at t = 0 :
[A]0 0, [B]0 = 0, [C]0 = 0
[A] = [A]0 e –k1t

d [B]
and solving = k1[A]0 e –k1t – k2[B] gives
dt
k 1[ A ] 0 k t k t
[B] = k e 1 e 2
2 k1
JPR\COMP.251\D\Allen(IIT-JEE Wing)\2020–21\Nurture\Che\Unit-08\Chemical Kinetics

To find [C], we use conservation of matter. The total number of moles present is constant with time, so
[A] + [B] + [C] = [A]0.

k2 k t k1 k t
Hence [C] = [A]0 k e 1 e 2
2 k1 k 2 k1
Figure shows plots [A], [B], and [C] for two values of k 2/k1.Note of maximum in the intermediate species [B].

191
JEE-Chemistr y

[A] 0 k2 = 6k1 [A]0 k2


1
k1
[A] 6
[A]
[C] [B]

Conc.
1 [A]
Conc.

1 [A] 0
2 0 2

[C]
[B]
0 0
0 0.5 1 1.5 0 2 4 6
k1 t k1 t
(a) (b)

Concentration versus time for the consecutive first-order reactions


1
A B C with rate constant k1 and k2. (a) k2 = 6k1; (b) k2 = k.
6 1
(iii) Reversible reaction : So far, we have neglected the reverse (or back) reaction, an assumption that is
strictly valid only if the equilibrium constant is infinite but that holds well during the early stages of a
reaction. We now allow for the reverse reaction.
Let the reversible reaction A C (with stoichiometric coefficients of 1) be first order in both the forward
(f) and back (b) directions, so that rf = kf [A] and rb = kb [C].
Then

d [ A] d[A] d [ A]
= + dt = – kf [A] + kb [C] = kb[C]0 + kb[A]0 – (kf + kb) [A]
dt dt f b
In the limit as t , the system reaches equilibrium, the concentration of each species is constant, and
d [ A] d [ A]
is 0. Let [A]eq be the equilibrium concentration of A. Setting = 0 and [A] = [A]eq , we get
dt dt
kb[C]0 + kb[A]0 = (kf + kb) [A]eq
d [ A]
Now = (kf + kb) [A]eq – [A]. Using the identity f (x + s) –1 dx = In (x + s) to integrate this equation,
dt

[ A ] [ A ]eq ( k f k b )t
we get In = (kf + kb)t or [A] – [A]eq = ([ A ]0 [ A ])e
[ A ]0 [ A] eq

A C

[A]/[A]0 kf/kb = 2
0.8

0.6
JPR\COMP.251\D\Allen(IIT-JEE Wing)\2020–21\Nurture\Che\Unit-08\Chemical Kinetics

0.4
[C]/[A]0
0.2

0
0 31 4 52
jt
1Concentrations versus time for the reversible first-order reaction A C with forward and reverse rate
constants kf and kb plotted for the case
kf/kb = 2. As t , [C]/[A] 2, which is the equilibrium constant for the reaction.

192
Chemical Kinetics

Illustration 33. A follows parallel path, first order reaction giving B and C as
k1
B

5A
k2
2C

If initial concentration of A is 0.25 M, calculate the concentration of C after 5 hours of reaction.


[Given : k1 = 1.5 × 10–5 s–1, k2 = 5 × 10–6 s–1]
2.303 [A]
Solution. k= log (k = k1 + k2 = 1.5 × 10–5 + 5 × 10–6 = 2 × 10–5 s–1)
t [A]t

2.303 0.25
2 × 10–5 = log
t [A]t
[A]t = 0.1744 M
[A]decomposed = [A]0 – [A]t = 0.25 – 0.1744 = 0.0756 M
6
k2 2 5 10 2
Fraction of C formed = × [A]decomped × = 5 × 0.0756 × = 7.56 × 10–3 M
(k1 k 2 ) 5 2 10 5
(5 moles of A are used to give 2 moles of C)

Based on Miscellaneous Reactions


1. The mechanism of the reaction 2NO + O2 2NO2 is :
k1 k2
NO + NO N2O2 (fast) ; N2O2 + O2 2NO2 (slow)
k–1
The rate constant of the reaction is
k1
(A) k2 (B) k2k1 (k–1) (C) k2k1 (D) k2 k
1

2. In the sequence of reaction :


k k2 k
A 1
B C 3 D, k3 > k2 > k1 then the rate determining step of the reaction is
(A) A B (B) C D (C) B C (D) A D

3. Consider this reaction. 2NO2(g) + O3(g) N2O5(g) + O2(g)


The reaction of nitrogen dioxide and ozone represented in first order in NO 2(g) and O3(g) & independent of O2.
Which of these possible reaction mechanisms is consistent with the rate law?
Mechanism I. NO2 + O3 NO3 + O2 slow
NO3 + NO2 N2O5 fast
JPR\COMP.251\D\Allen(IIT-JEE Wing)\2020–21\Nurture\Che\Unit-08\Chemical Kinetics

Mechanism II. O3 O2 + O fast


NO2 + O NO3 slow
NO3 + NO2 N2O5 fast
(A) I only (B) II only (C) both I and II (D) neither I nor II

4. A hypothetical reaction, A2 + B2 2 AB follows the mechanism as given below :


A2 A+A ...(fast)
A + B2 AB + B ... (slow)
A+B AB ...(fast)
The order of the overall reaction is :
1
(A) 2 (B) 1 (C) 1 (D) zero
2
193
JEE-Chemistr y

5. For the reaction, A + B C + D. The variation of the concentration of the products is given by the curve :

Y
Z

Conc.
W
X
Time

(A) X (B) Y (C) Z (D) W

6. A substance undergoes first order decomposition. The K1 B


decomposition follows two parallel first order reactions as : K1=1.26 × 10–4sec–1
The percentage distribution of B and C are : A and K2=3.8 × 10–5sec–1
(A) 80% B and 20% C (B) 76.83% B and 23.17% C K2
(C) 90% B and 10% C (D) 60% B and 40% C C

7. A gaseous compound A reacts by three independent first order processes (as shown
in figure) with rate constant 2 × 10–3, 3 × 10–3 and 1.93 × 10–3 sec–1 for products B, C and D respectively.
If initially pure A was taken in a closed container with P = 8 atm, then the partial pressure of B (in atm) after
100 sec from start of experiment :

k1 B(g)
k2
A(g) C(g)
k3
D(g)
(A) 0.288 (B) 0.577 (C) 1.154 (D) none of these

18.0 RADIOACTIVITY
AL

The spontaneous emission of particles, electromagnetic radiation or both by unstable nuclei, is known as
radioactivity.
PROPERTIES OF , -PARTICLES AND -RAYS.
Properties Alpha beta Gamma
1. Nature Fast moving He Fast moving High energy
nuclei electrons

2. Representation 2
He4 or –1
e0 or –1 0
or 00
3. Charge 2 unit (+ve) 1 unit (–ve) no charge
4. Velocity 1/10 of light 33% to 90% of Same as light
light waves
5. Relative penetrating 1 or (0.01 mm of 100 or (0.1 cm 10000 or (8 cm lead
JPR\COMP.251\D\Allen(IIT-JEE Wing)\2020–21\Nurture\Che\Unit-08\Chemical Kinetics

power Al foil Al foil or 25 cm steel


6. Travel distance 2 – 4 cm 200 – 300 cm 500 m
in air
7. Kinetic energy high low –
8. Effect on ZnS plate Luminosity Little effect –
9. Mass g/particle 6.65 × 10–24 9.11 × 10–24 –
10. Relative ionising power 10000 100 1

194
Chemical Kinetics
Kinetics of nuclear distintegration
dN
Radioactive decay is a first order process. Hence = N or N = N0 e t
dt
where N = number of radioactive nuclei at any time t ;
N0= number of radioactive nuclei at t = 0 ;
= decay constant.
dN
Activity activity (A) = = N
dt
S.I. units :
Disintegration per second (symbol s 1 or dps). 1 dps = 1 Bq (Becquerel)
Other units:
1Ci (Curie) = 3.7 × 1010dps.
1 Rd (Rutherford) = 106 dps
Specific activity = dps / gm

Half life (t½) The time taken by half the nuclei (originally present) to decay. t ½ = 0.693/

1
Note : After n half lives have passed, activity is reduced to of its initial value.
2n

Average life (t av)

1 t1/ 2
Tavg = = = 1.44 t1/2
0.693
Application of Nuclear disintegration :
(i) Carbon dating (ii) Rock dating

210
Illustration 34. 84
Po decays with emission of -particle to 206
82
Pb with a half-life period of 138.4 days. If 1g of 210
84
Po
is placed in a sealed tube, how much helium will be accumulated in 69.2 days ? Express the answer
in cm3 at STP.
210
Solution. Po 206
84 82
Po + 24He
Amount of 21084
Po left after 69.2 days can be calculated by applying
N = N0 (1/2)n
69.2 1
n = t/t1/2 =
138.2 2
1/ 2
1
N=1× = 0.702 g
2
JPR\COMP.251\D\Allen(IIT-JEE Wing)\2020–21\Nurture\Che\Unit-08\Chemical Kinetics

Amount of polonium disintegrated = 1 – 0.7072 = 0.2928 g


0.2928
Moles of polonium in 0.2928 g =
210
0.2928
Moles of helium atoms formed =
210
0.2928
Volume of helium collected = × 22400 = 31.23 cm3
210

195
JEE-Chemistr y
18.1 Carbon Dating
AL

The cosmic ray generates neutrons in the atmosphere which bombards the nucleus of atmospheric nitrogen to
form radioactive 14C hence 14C in the atmosphere has been remaining constant over thousands of years. In living
materials, the ratio of 14C to 12C remains relatively constant. When the tissue in an animal or plant dies, assimilation
of radioactive 14C ceased to continue. Therefore, in the dead tissue the ratio of 14C to 12C would decrease depending
on the age of the tissue.
14
7
N + 10n 14
6
C + 11p
14 14
6
C 7
N + –10e
A sample of dead tissue is burnt to give carbon dioxide and the carbon dioxide is analysed for the ratio of 14C
to 12C. From this data, age of dead tissue (plant or animal) can be determined.

2.303 N0 2.303×t1/2 (14 C) N0


Age (t) = log Age = log
N 0.693 N
N0 = ratio of 14C/12C in living plant
N = ratio of 14C/12C in the wood

2.303×t1/2 A0
Age = log
0.693 A
n
1 t
A0 = Original activity A = Final activity. Also, N = N0 where n = t
2 1/ 2

18.2 Rock Dating


AL

It is based on the kinetics of radioactive decay. It is assumed that no lead was originally present in the sample
and whole of it came from uranium.
Initial no. of mole (N0) = [U] + [Pb]
Final no. of mole (N) = [U]
N0 [U] [Pb] [Pb] 2.303 N0
=1+ ; t= log
N [U] [U] N

2.303 [Pb] [Pb] t


t= log 1 Also, 1 = (2)n n
[U] [U] t1/ 2

Illustration 35. A sample of uranium mineral was found to contain 206Pb and 238U in the ratio of 0.008 : 1. Estimate
the age of the mineral (half-life of 238U is 4.51 × 109 years).
206
2.303 Pb
Solution. t= log 1 238
U
JPR\COMP.251\D\Allen(IIT-JEE Wing)\2020–21\Nurture\Che\Unit-08\Chemical Kinetics

206
2.303 Pb
t= t1/ 2 × log 1 238 Ratio by mass 206
Pb : 238
U = 0.008 : 1
0.693 U

0.008 1
Ratio by moles 206Pb : 238U = : = 0.0092
206 238

2.303 4.51 109


t= log (1 + 0.0092)
0.693

2.303 4.51 109 0.0412


109 = 5.945 × 107 years
= × 0.00397 =
0.693 0.693

196
Chemical Kinetics

18.3 Stability of Nuclei with Respect to Neutron - Proton Ratio


AL

If number of neutrons is plotted against the number of protons, the 140

Number of neutrons (n)


120
stable nuclei lie within well-defined region called zone of stability. All 100 Zone of
stability
the nuclei falling outside this zone are invariably radioactive and unstable 80 Unstable
region Unstable
region
in nature. Nuclei that fall above the stability zone has an excess of 60
40
neutrons while those lying below have more protons. 20 n/p=1
These nuclei attain stability by making adjustment in n/p ratio. 0
20 40 60 80 100 120 140
Number of protons (p)
When (n/p) ratio is higher than that required for stability
Such nuclei have tendency to emit -rays (transforming a neutron into proton).
1 1 0
0
n 1
p + –1 e ( -particle)
14 14 0
6
U 7
N + –1e
n
p
1.33 1
87 87 0
36
Kr 37
Rb + –1 e
n 51 50
:
p 36 37

When (n/p) ratio is lower than that required for stability


Such nuclei have tendency to increase n/p ratio by adopting any of the following three ways.
By emission of an -particle (natural radioactivity).
298 234
92
U 90
Th + 42He ( -particle)
n 146 144
P 92 90
= 1.50 = 1.60
By emission of positron
13 13 0
7
N 6
C ++1 e
n 6 7
p 7 6
By K-electron capture
194
79
Au + e0–1 194
78
Pt
n 115 116
P 79 78
-emission is usually observed in natural radioactive isotopes while emission of positron or K-electron capture
is observed in artificial radioactive isotopes. The unstable nuclei continue to emit or -particle until stable
nucleus comes into existence.
18.4 Nuclear Fission
JPR\COMP.251\D\Allen(IIT-JEE Wing)\2020–21\Nurture\Che\Unit-08\Chemical Kinetics

AL
It is a nuclear reaction in which heavy nucleus splits into lighter nuclei of comparable masses with release of
large amount of energy by bombardment with suitable sub-atomic particles, i.e.
140 93 1
Ba + 36Kr + 30n
56

1
0
n + 235
92
U Xe144 + 38Sr 90 + 20n1
54

144 90 1
Cs + 37Rb + 20n
55

If the neutrons from each nuclear fission are absorbed by other 92U235 nuclei, these nuclei split and release even
more neutrons. Thus, a chain reaction can occur. A nuclear chain reaction is a self sustaining series of nuclear
fissions caused by the previous neutrons released from the previous nuclear reactions.

197
JEE-Chemistr y

0 n1 235
92 U
1
56 Ba140 +0 1n 235
92 U 0 n 235
92 U
n 1 235
U
1
0n + 235
92 U
+01n 92 U235 0 92

36 Kr93+ 01n 92 U235


A nuclear fission reactor is a devicve that permits a controlled chain nuclear fissions. Control rods made of
elements such as boron and cadmium, absorb additional neutrons and can therefore, slow the chain reactions.

18.5 Nuclear Fusion


AL

It is a nuclear reaction in which two lighter nuclei are fused together to form a heavier nuclei. To achieve this,
colliding nuclei must posses enough kinetic energy to overcome the initial force of repulsion between the positively
charged core. At very high temperature of the order of 106 to 107 K, the nuclei may have the sufficient energy
to overcome the repulsive forces and fuse. Such reactions are therefore also known as thermonuclear reactions.
2
1H + 13H ¾¾® 42 He + 10 n + 17.8MeV 2
1H + 12H ¾¾® 42 He + 24.9 MeV
1
1H + 13H ¾¾® 42 He + 20.0 MeV 7 1
3 Li + 1H ¾¾® 42 He + 17.7 MeV

Radioactivity
1. A piece of wood from an archeological source shows a C-14 activity which is 60% of the activity found today.
Calculate the age of the sample ( t 1 for 14
6C = 5770 years) :
2

(A) 3515 years (B) 300 years (C) 4253 years (D) 4000 years

2. A radioactive isotope x with half life of 1.37 × 109 years decays to y which is stable. A sample of rock from
the moon was found to contain both the elements x and y in the ratio 1 : 7. What is the age of the rocks :
(A) 1.96 × 108 years (B) 3.85 × 109 years (C) 4.11 × 109 years (D) 3.06 × 109 years

3. How many a and b particle will be emitted when ac X changes to bd Y ?

a-b (a - b) b-a (a - b)
(A) a = , b=d+ -c (B) a = , b=d+ -c
4 2 4 2
b-a (a + b) b+a (a + b)
(C) a = , b=d+ +c (D) a = , b=d+ +c
4 2 4 2

4. What is the a-activity in disintegration per minute 1 g sample of 226Ra. (t1/2 = 1620 year)
(A) 1.08 × 106 events/min (B) 2.16 × 1012 events/min
6
(C) 1.28 × 10 events/min (D) 1.08 × 1010 events/min
JPR\COMP.251\D\Allen(IIT-JEE Wing)\2020–21\Nurture\Che\Unit-08\Chemical Kinetics

5. The half life of the nuclide Rn220 in 54.5 sec. What mass of radon is equivalent to 1 millicurie.
(A) 1.06 × 10–15 kg (B) 1.06 × 1015 kg (C) 2.06 × 1010 kg (D) 2.06 × 10–10 kg
210
6. 84Po decays with a to 206
82Pb with a half life of 138.4 days. If 1.0 g of Po
210
is placed in a closed tube, what
volume of helium will be accumulated in 69.2 days at STP.
(A) 22.4 ml (B) 11.2 ml (C) 31.2 ml (D) 48.2 ml
218
7. 84Po (t1/2 = 3.05 min) decay to 82Pb214 (t1/2 = 2.68 min) by a-emission, while Pb214 is a b-emitter. In an
experiment starting with 1 g atom of pure Po218, how much time would be required for the number of nuclei of
214
82Pb to reach maximum.
(A) 3.15 min (B) 4.12 min (C) 5.15 min (D) 5.30 min

198
Chemical Kinetics

Expression for rate constants for reaction of different orders.


Type of reaction Integrated rate Unit of rate Half-life t3/4 life
equation constant period period
d[A] a
Zero order reaction k 0 [A]0 Concentration time–1 t 1= --
dt 2
2k 0

dx
=k
dt
2.303 a 0.693 0.693 1.382
First order reaction k1 = log 10 time–1 t 1= t 3 =2× =
t (a–x) 2
K1 4
k1 k1

2.303 b(a-x) 1 3
Second order reaction k 2= log Mole–1litre time–1 t1 t 3=
t(a-b) a(b-x) 2
k 2a 4
k 2a

dx
=k(a-x)2
dt
x×(2a-x) 3
Third order reaction k 3= Litre2mole–2time–1 t 1= --
t2a 2 (a-x)2 2
2k 3a 2

dx
=k(a-x)3
dt

Examples of reactions of 1st order and their formula for rate constants
2.303 V
(i) 2N2O5 4NO2 + O2 ; k = log V V
t t
where V = volume of O2 gas collected at infinite time
Vt = volume of O2 gas collected at time t

2.303 V
(ii) NH4NO2 2H2O + N2 ; k = log V V
t t
where V and Vt are volumes of N2 gas collected after infinity time and after time t respectively.

1 2.303 VD
(iii) H2O2 H2O + O2 ; k = log V
2 t t
where V0 and Vt are the volumes of KMnO4 solution used for titrating a definite volume of the
reaction mixture at t = 0 and at time t respectively.

(iv) CH3COOC2H5 + H2O CH3COOH + C2H5OH


JPR\COMP.251\D\Allen(IIT-JEE Wing)\2020–21\Nurture\Che\Unit-08\Chemical Kinetics

2.303 V V0
k= log V Vt
t
where V0, Vt and V are the volume of NaOH solution used for titration mixture at zero time,
after time t and after infinity respectively.

H
(v) C12H22O11 + H2O C6H12O6 + C6H12O6
Glucose Fructose
2.303 (r0 r )
k= log (r r )
t t
where r0, rt and r are the polarimetric reading at zero time, after time t and after infinity respectively.

199
JEE-Chemistr y

Rate law equation for reactions involving parallel reaction

k1
B
d[A]
A Rate = – [dt] = k1 [A] + k2 [A] = [k1 + k2] [A]
k2
C

Degree of dissociation at any time t = (1 – e–kt)

JPR\COMP.251\D\Allen(IIT-JEE Wing)\2020–21\Nurture\Che\Unit-08\Chemical Kinetics

200
Chemical Kinetics

SOME WORKED OUT EXAMPLES


Illustration 1.
Write the units of the rate constants for a (i) Zeroth order, (ii) half order, (iii) first order, (iv) 3/2 order, (v) second
order, (vi) 5/2 order, (vii) third order reactions.
Solution.
Unit of Rate Constant = (Mole)1-n (Litre)n-1 Sec-1 Where n is the order of Reaction
(i) For Zeroth order = Mole(1-0) (Litre)(0-1) Sec-1
Unit of K
n=o Mole Litre(-1) Sec-1
Similarly For others

Illustration 2.
The reaction CO(g) + NO2(g) CO2 + NO is second order in NO2 and zero order in CO at temperatures less
than 500K.
(a) Write the rate expression for the reaction.
(b) How will the reaction rate change if the NO2 concentration is halved?
Solution.
d d
(a) [CO] = (NO2) = K [NO2]2 Order is zero w.r.t. CO but Conc will Still change
dt dt
(b) Rate of Rxn = K[NO2 ]2 of Conc of NO2 Half The Rate becomes One fourth

Illustration 5.
For a reaction A + 3B Product, Rate = {– d[A] / dt} = k [A] 2 [B] , the expression for the rate of reaction in
terms of change in the concentration of B; {– d[B] / dt} will be :
(A) k[A]2 [B] (B) k [A]2 [3B] (C) 3k [A2] [B] (D) (1/3) k [A2] [B]
Ans. [C]
Solution.
For the given reaction
d[ A ] 1 d[B] d[B]
= = K[A]2[B] Then = 3K[A]2[B]
dt 3 dt dt

Illustration 3.
For the reaction 2N2O5 (g) 4NO2 (g) + O2 (g) the concentration of NO2 increases
by 2.4× 10–2 Mol lit–1 in 6 seconds. Calculate the rate of appearance of NO2 and the rate of disappearance
of N2O5 -
(A) 4 × 10–3 mol lit –1 sec –1, 2 × 10–3 mol lit –1 sec –1
(B) 2 × 10–3 mol lit –1 sec –1, 4 × 10–3 mol lit –1 sec –1
(C) 3 × 10 –3 mol lit –1 sec –1, 3 × 10–3 mol lit –1 sec –1
(D) None of these
Ans. [A]
Solution.
JPR\COMP.251\D\Allen(IIT-JEE Wing)\2020–21\Nurture\Che\Unit-08\Chemical Kinetics

1 [N2O5 ] 1 [NO2 ]
Rate of reaction, – =
2 t 4 t
Since, NO2 is the product, therefore, its concentration when t = 0 is zero.
Rate of appearance of NO2 i.e.
2
[NO2 ] 2 . 4 10
= = 4 × 10–3 mol lit–1 sec–1
t 6
1 [NO2 ]
Thus, rate of reaction =
4 t
3
4 10
= mol lit –1 sec –1 = 1 × 10–3 mol lit –1 sec –1
4
201
JEE-Chemistr y

Rate of diapearance of N2O5


[N2O5 ]
i.e. = 2 × Rate of reaction
t
= 2 × 1 × 10–3 mol lit–1 sec –1
= 2 × 10–3 mol lit–1 sec–1

Illustration 4.
For the chemical reaction 3O2 — 2O3, the rate of formation of O3 is 0.04 mole lit–1 sec–1. Determine
the rate of disappearance of O2 -
(A) 0.04 mol lit –1 sec –1 (B) 0.08 mol lit –1 sec –1
(C) 0.10 mol lit –1 sec –1 (D) 0.06 mol lit –1 sec –1
Ans. [D]
Solution.
d [O3 ]
= 0.04 mol lit–1 sec —1
dt
We know, Rate
1 d [O2 ] 1 d [O3 ]
= – =
3 dt 2 dt
d [O2 ] 3 d [O3 ] 3
– = × = × 0.04 = 0.06 mol lit–1 sec–1.
dt 2 dt 2

Illustration 5.
For the reaction
2N2O5(g) — 4NO2 (g) + O2(g), the concentration of NO2 increases by 2.4 × 10–2 Mol lit.–1 in 6 second.
What will be the rate of appearance of NO2 and the rate of disappearance of N 2O5
(A) 2 × 10 –3 mol lit. –1 sec –1 , 4 × 10–3 mol lit.–1 sec –1
(B) 4 × 10 –3 mol lit. –1 sec –1 , 2 × 10–3 mol lit.–1 sec –1
(C) 2 × 10 –3 mol lit. –1 sec –1 , 2 × 10–3 mol lit.–1 sec –1
(D) None of these
Ans. [B]
Solution.
1 [N2O5 ] 1 [NO2 ]
Rate of reaction = – =
2 t 4 t
Since NO2 is the product, therefore, its concentration when t = 0 is zero.
Rate of appearance of NO2 i.e.
[NO2 ] 2 .4 10 2
=
t 6
= 4 × 10–3 mol lit.–1 sec–1
1 [NO2 ]
JPR\COMP.251\D\Allen(IIT-JEE Wing)\2020–21\Nurture\Che\Unit-08\Chemical Kinetics

Thus, rate of reaction =


4 t

4 10 3
= mol lit.–1 sec –1
4
= 1 × 10–3 mol lit –1 sec –1.
[N2O5 ]
Rate of disappearance of N2O5 i.e. = 2 × Rate of reaction
t
= 2 × 1 × 10–3 mol lit. –1 sec –1
= 2 × 10–3 mol lit.–1 sec–1.

202
Chemical Kinetics
Illustration 6.
Following reaction was carried out at 300 K. 2SO2(g) + O2 (g) —— 2SO3(g)
How is the rate of formation of SO3 related to the rate of disappearance of O2
[O 2 ] 1 [SO3 ] [O 2 ] [SO3 ]
(A) – = + (B) – =
t 2 t t t
[O 2 ] 1 [SO3 ]
(C)– = – (D) None of these
t 2 t
Ans. [A]
Solution.
Rate of reaction
1 [SO2 ] [O 2 ] 1 [SO3 ]
= – = – = +
2 t t 2 t
Therefore, rate of disappearance of O2 is related to rate of formation of SO3 as -
[O 2 ] 1 [SO3 ]
– = +
t 2 t

Illustration 7.
For the reaction 2NO (g) + H2(g) — N2O (g) + H2O (g), at 900 K. following data are observed.

Initial pressure Initial pressure


Rate
of NO (atm) of H2 (atm)

0.15 0.40 0.02


0.075 0.40 0.005
0.150 0.02 0.10

Find out the rate law and order of reaction -


2 1 2 1
(A) k [PNO2 ] [PH2 ] , 3 (B) k [PNO2 ] [PH2 ] , 2
2 1 2 1
(C) k [PNO2 ] [PH2 ] ,1 (D) k [PNO2 ] [PH2 ] ,0
Ans. [A]
Solution.
Rate law equation in terms of pressure is
m n
Rate = k [PNO2 ] [PH2 ]
Substituting the given data in rate law equation we get,
(i) 0.02 = (0.15)m [0.4]n
(ii) 0.005 = (0.075)m [0.4]n
(iii) 0.010 = (0.15)m [0.2]n
Divide equation (i) by (ii)

0.02 k [0.15]m [0.4]n


JPR\COMP.251\D\Allen(IIT-JEE Wing)\2020–21\Nurture\Che\Unit-08\Chemical Kinetics

=
0.005 k [0.075]m [0.4]n
or 4 = [2]m m = 2
Divide equation (i) by (iii)

0.02 k [0.15]m [0.4]n


=
0.01 k [0.15]m [0.2]n
or 2 = (2)n n = 1

203
JEE-Chemistr y

2 1
Rate law, r = k [PNO2 ] [PH2 ]
Order of reaction = 2 + 1 = 3

Illustration 8.
A study of chemical kinetics of the reaction A + B Products, gave the following data at 25ºC.
Experiment [A] [B] d [Products]dt
1 1.0 0.15 4.20 × 10–6
2 2.0 0.15 8.40 × 10–6
3 1.0 0.20 5.60 × 10–6
Calculate the rate law -
(A) K [A] (B) K [B] (C) K [A] [B] (D) None of these
Ans. [C]
Solution.
Doubling the concentration of A increases the rate by two times. Raising the concentrating of B by 4/
3 times increases the rate of B by 4/3 times. Thus the reaction is of first order with respect to both A
and B. So rate law = K [A] [B].

Illustration 9.
In the reaction of NO by H2 it was observed that equimolecular mixture of gases at 500 mm was half
changed in 100 seconds. In another experiment with an initial pressure of 250 mm, the reactions was
half completed in 200 seconds. Calculate the order of reaction-
(A) 1 (B) 2 (C) 3 (D) 4
Ans. [B]
Solution.

Since, t1 / 2
1
, So
t1 LM a OP
2
n 1
LM p OP
2
n 1

an 1 t2 = Na Q
1
=
Np Q
1

Here p1 = 500 mm, p2 = 250 mm,


t1 = 100 sec, t2 = 200 sec. Thus
n 1
100 250
= or n = 2
200 500
i.e. Order of reaction = 2nd .

Illustration 10.
For a reaction 3A products, it is found that the rate of reaction doubles, if concentration of A is
increased four times, calculate order of reaction
(A) 1 (B) 1/2 (C) 2 (D) 3
Ans. [B]
Solution.
Rate = k [Reactant]n
if [Reactant] = a ; rate = r1
JPR\COMP.251\D\Allen(IIT-JEE Wing)\2020–21\Nurture\Che\Unit-08\Chemical Kinetics

if [Reactant] = 4a ; rate = 2r 1 r1 = k [4a]n


n
2r1 = k [4a]
n
1 1 1
= n =
2 4 2

Illustration 11.
A first order reaction was started with a decimolar solution of the reactant. After 8 minutes and 20
seconds, its concentration was found to be M/100. Determine the rate constant of the reaction
(A) 4.6 × 10–3 sec –1 (B) 16.6 × 10–3 sec –1
(C) 24.6 × 10–3 sec –1 (D) 40.6 × 10–3 sec –1
Ans. [A]
204
Chemical Kinetics

Solution.
Here a = 0.1 M
M
a – x = = 0.01 M
100
t = 8 minutes 20 seconds
= 500 seconds
Substituting the values in the first order reaction.
2 . 303 a
K = log
t a x

2 . 303 0 .1
= log 0 . 01
500

2 . 303
= log 10
500
4 . 606
= × 1
1000
= 4.6 × 10–3 sec–1

Illustration 12.
The first order rate constant for the decomposition of N2O5 is 6.2 × 10–4 second–1. What will be the
half life period in second for this decomposition.
(A) 1117.74 seconds (B) 117.74 seconds (C) 1177.74 seconds (D) None
Ans. [A]
Solution.
We know that for a first order reaction,
0 . 693
t1/2 =
K
0 . 693
t1/2 = 6 2 10 4
.
= 1117.74 seconds

Illustration 13.
In a first order reaction A Products, the ratio of a and (a – x) was found to be 8 after 60 minutes.
Calculate the rate of the reaction in moles of A reacted per minute, If its concentration is 0.1 mole litre–1-
(A) 5.566 × 10–3 mole litre–1 min–1 (B) 3.466 × 10–3 mole litre min
–3 –1 –1
(C) 4.366 × 10 mole litre min (D) 3.466 × 10–3 mole litre–1 min–1
Ans. [D]
Solution.
2.303 a 2.303
K= log = log 8
t a x 60
JPR\COMP.251\D\Allen(IIT-JEE Wing)\2020–21\Nurture\Che\Unit-08\Chemical Kinetics

= 3.466 × 10–2 min–1


dx
Rate of reaction, = K[A]
dt
= 3.466 × 10–2 × 0.1= 3.466 × 10–3 mole litre–1 min–1

Illustration 14.
A second order reaction with two reactants is started with 0.1 m concentration of each reactant. It is 20%
complete in 500 seconds. How much time it take to go 60% completion
(A) 5000 seconds (B) 3000 seconds (C) 2000 seconds (D) 4000 seconds
Ans. [B]

205
JEE-Chemistr y
Solution.
1 x
K2 = × .
t a(a x)
Since a = 0.1 so for 20% completion x would be 20% of initial concentration.
i.e. a = 0.1
20 1 0.02 1
x = × 0.1 = 0.02 K2 = × 01
. (01
. 0.02) =
100 500 200
For 60% completion,
60
x = × 0.1 = 0.06
100
1 0 . 06 1 1 0 . 06
K2 = × 0 .1 (0 .1 0 . 06) or = × 0 .1 (0 .1 0 . 06) or t = 3000 sec.
t 200 t

Illustration 15.
The formation in water of d potassium chromo oxalate from its l form is reversible reaction which is first order
in both directions, the racemate being the equilibrium product. A polarimeter experiment at 22 0C showed that,
after 506 sec, 12 mole % of the l isomer was converted to the d form. Find the rate constant for the forward
and the reverse reactions.
Ans. Kf = kb = 2.71 x 10- 4 sec -1
Solution.
– potassium chromo - oxalate d potassium chromo oxalate
t=0 a 0
t=teq. (a – xeq.) xeq.

Kf x eq.
Keq. = K = a – x eq.
b

a
As xeq. = ; Kf = Kb
2

2 .303 x eq.
(Kf + Kb) = log x – x
t eq.

2 .303 0.5a
2Kf = log 0.5a – 0 .12a
560
2Kf = 5.42 × 10–4
Kf = Kb = 2.71 × 10–4

Illustration 16.
The values of rate constant for the decomposition of N2O5,
1
N2O5 N2O4 + O
2 2
are 3.50 x 10—5 and 5 x 10—3 at 27ºC and 67ºC, respectively. Calculate the energy of activation -
JPR\COMP.251\D\Allen(IIT-JEE Wing)\2020–21\Nurture\Che\Unit-08\Chemical Kinetics

(A) 14.8 k Cal/Mol (B) 24.8 k Cal/Mol


(C) 28.8 k Cal/Mol (D) 34.8 k Cal/Mol
Ans. [C]
Solution.
Given
k2 = 5 x 10—3 , T2 = 67 + 273 = 340 K
k1 = 3.50 x 10—5 , T1 = 27 + 273 = 300 K

k2 Ea T2 T1
R = 2 × 10–3 kcal mol–1 k–1 log k = x T1T2
1 2.303 xR

206
Chemical Kinetics

3 340 300
5 10 E a
2.303 log 5 = 340 x 300
3.5 10 R
Ea = 25 kcal/Mole

Illustration 17.
The reaction
A+B products
is first order with respect to both A and B has a rate constant of 6.0 mol–1 sec–1. at 27°C. Calculate the initial
rate of the reaction at 47°C when equal volumes of A and B of concentration 0.02 moles litre –1 in each are
mixed. The activation energy of the energy of the reaction is 42 kJ mol –1 .
Solution.
Reaction A + B Product

K2 E 1 1
We know log K1 = T1 T2
2.303R

K2 E T2 T1
log K1 = T1T2
2.303R

K2 42 10 3 [320 300] 840


log = =
6 2.303 8.3 300 320 1835 .03

K2
log = .4577
6

K2
= anti log (.4577)
6
K2 = 2.863 × 6 = 17.178
Rate at 47oC will be
The Rate2 = 17.178 × [0.01] × [0.01]
Rate2 = 17.178 × 10-4 = 1.7178 × 10-3

Illustration 18.
In Arrhenius`s equation for a certain Reaction, the value of A and E (activation energy) are 6 × 10 13 s-1 and
98.6 kJ mol-1 respectively. If the reaction is of first order, at what temperature will its half-life period be 20
minutes ?
Solution.
k = Ae-E/RT
E
In k = In A –
RT
JPR\COMP.251\D\Allen(IIT-JEE Wing)\2020–21\Nurture\Che\Unit-08\Chemical Kinetics

E
2.303 log k = 2.303 log A –
RT
E
or log k = log A – . ...(i)
2.303RT
Given that A = 6 × 10 s , E = 98.6 kJ mol-1
13 -1

t1/2 = 20 × 60 s.
0.6932 0.6932 -1
For first-order reaction k = t1/ 2 = 1200 s
Thus (i) becomes,

207
JEE-Chemistr y

0.6932 98.6
log = log (6 × 1013) - 3 [R = 8.314 × 10-3 kJ/K/mol]
1200 2.303 8.314 10 T
T = 302.26 K.

Illustration 19.
For a gaseous reaction A products, the half-life of the first order decomposition at 400 K is 150 minutes
and the energy of activation is 65.0 kJ mole –1. What fraction of molecules of A at 400 K have sufficient energy
to give the products ?
Solution.
K
= e–Ea/RT = 3.13 × 10-6. = Fraction of Molecule Having Sufficient Energy
A

Illustration 20.
The following data is for the decomposition of ammonium nitrite in aqueous solution.

Volume of N2 in cc. Time (minutes)


6.25 10
9.00 15
11.40 20
13.65 25
35.05 infinity

The order of the reaction is


Solution.
NH4NO2(s) N2(q) + 2H2O(l)
Let Vt be the volume of N2 Collected at time ‘t’
V = be the volume of N2 Collected at the end of the Reaction
a v
(a - x) V - Vt
Then from the given data We assume the Rxn is first order Then
2.303 a 2.303 V
K= log = log
t (a x ) t V – Vt

2.303 35.05
From Ist data K = log = 1.96 × 10–2 sec–1
10 (35.05 6.25 )
2.303 35.05
IInd data K= log = 1.96 × 10–2 sec–1
15 (35.05 9)
2.303 35 .05
IIIrd data K= log = 1.96 × 10–2 sec–1
20 (35.05 11.40 )
From these relation the value of K are same the Rxm will be First order

Illustration 21.
D
A B+C
JPR\COMP.251\D\Allen(IIT-JEE Wing)\2020–21\Nurture\Che\Unit-08\Chemical Kinetics

Time 0 t
Volume of reagent V1 V2 V3

The reagent reacts with only B, C and D. Find k.


Solution.
D
A B+ C
at t=o a o o
t=t (a - x) x x
t= o a a
at t = 0 only D. React

208
Chemical Kinetics
after t = 0 reagent react with B.C.D.
2a a (V3 - V1)
( V3 - V1 )
aa
2
2x a (V2 - V1)
2(a - x) a (V3 - V1 - V2 + V1)
( V3 - V2 )
(a - x) a
2

1 ( V3 - V1 )
Then K = ln ( V - V )
t 3 2

Illustration 22.
k1 k2
In the case of first order consecutive reactions A ¾¾ ® B ¾¾
¾® C (Product), one may show that the
concentration of B is given by

k1C0A k k
CB = é e - t - e - t ùû .
1 2

(k 2 - k1 ) ë
The usual assumptions are CA = CA0 at t = 0 while CB = CC = 0 at t = 0. Show that CB has its maximum value
æ 2.303 ö æ k2 ö
at time t = çç k - k ÷÷ log çç ÷÷ .
è 2 1ø è k1 ø
Solution.

dCB k1C°A
= é k e - k 2 t - k 1e - k 1t ùû
dt (k 2 - k1 ) ë 2

equating this zero k 2 e -k 2 t = k1 e -k1 t

k2 æ k2 ö æ 2.303 ö æ k2 ö
\ = (k 2 -k 1 ) t \ln çç ÷÷ = (k – k ) t \ t = çç k - k ÷÷ log çç k ÷÷
k1 e
è k1 ø 2 1
è 2 1ø è 1ø

Illustration 23.
dN 1
Starting from the equation for radioactive decay i.e. – = lN, derive the expression, mean life = .
dt l
Solution.
dN
– = lN . This on integration gives (with N = N0 , at t = 0) ; N = N0 e–lt
dt

¥ ¥
é ¥ ù
ê
ò tdN ú ò t
dN
dt ò ( -lN0 )e -lt t dt
JPR\COMP.251\D\Allen(IIT-JEE Wing)\2020–21\Nurture\Che\Unit-08\Chemical Kinetics

ê ú dt t =0 1 1
t=0 t =0
Mean life = ê ú = = ¥ = . \ Mean life =
ê ¥
ú ¥ l l
ê
ê ò dN ú
ú
ò
dN
dt
dt ò
t =0
( -lN0 )e -lt dt
ë t=0 û t=0

Illustration 24.
The half life time for the decomposition of a substance dissolved in CCl4 is 2.5 hour at 30ºC. How much
of the substance will be left after 10 hours, if the initial weight of the substance is 160 gm
(A) 20 gm (B) 30gm (C) 40 gm (D) 10 gm
Ans. [D]
Solution.
No. of half life periods
209
JEE-Chemistr y

Total time 10
= = = 4
Time of one half life period 2.5
Now we know that the quantity left after ‘n’ half-life periods
= (1/2)n × Initial quantity
= (1/2)4 × 160
160
= = 10 gm
2 2 2 2

Illustration 25.
(a) The decomposition of HI to yield H2 and I 2 at 508°C has a half-life of 135 minutes when
PHI = 0.1 atm (initially), which comes down to one tenth of that value when the initial pressure is 1 atm.
Calculate the rate constant.
(b) In a kinetic study of the reduction of nitric oxide with hydrogen, the initial pressure of 340 mm, an
equimolar mixture of gases was reduced to half the value in 102 seconds. In another experiment, the
initial pressure of 288 mm, under the same conditions was reduced to half the value in 140 sec. Calculate
the order of the reaction.
Ans. (a) 0.074 atm–1 min–1 , (b) 3
Solution. (a) t1/2 = 135 minutes at PHI = 0.1 atm
t 1/2 = 13.5 minutes at PHI = 1 atm.
1
t1/2 PHI

Therefore order of the reaction, n = 2.


1 1 1
Rate constant, k =
t a x a

a
At t = t1/2 , (a – x) =
2

1 1
k = at = = 0.074 atm–1 min–1
1/ 2 0.1 135
(b) The reaction is not first order because the t 1/2 varies with initial concentration.

1 1
For a second order reaction, t1/2 is proportional to n 1
or t1/2 is proportional to
a a
where n = 2 and a = initial concentration
t1/2 × a = constant
from the given data, 340 × 102 = 34680
288 × 140 = 40320
The products are not constant, therefore the reaction is not second order also,
( t1/ 2 )1 ( an 1)
2
JPR\COMP.251\D\Allen(IIT-JEE Wing)\2020–21\Nurture\Che\Unit-08\Chemical Kinetics

For a third order reaction = n 1 1, 2 represent the first and second data given.
( t1/ 2 )2 (a )1
n 1
102 ( 288 )n 1
288
= =
140 ( 340 )n 1
340

log 0.7286 = (n – 1) log 0.8471


– 0.1375 = – (n – 1) 5 0.0721
0.1375
(n – 1) = = 1.91
0.0721
n = 1 + 1.91 = 2.91 3

* * * * *
210
Chemical Kinetics

ANSWERS
BEGINNER'S BOX-1
1. (D) 2. (B) 3. (A) 4. (C) 5. (B)
6. (B) 7. (C) 8. (C) 9. (C) 10. (C)

BEGINNER'S BOX-2
1. (A) 2. (A) 3. (A) 4. (B) 5. (B)
6. (C) 7. (A) 8. (A) 9. (D)

BEGINNER'S BOX-3
1. (D) 2. (C) 3. (B) 4. (A) 5. (C)
6. (D) 7. (C)

BEGINNER'S BOX-4
1. (D) 2. (A) 3. (A) 4. (C) 5. (B)
6. (B) 7. (C)

BEGINNER'S BOX-5
1. (C) 2. (C) 3. (A) 4. (B) 5. (A)
6. (C) 7. (B)
JPR\COMP.251\D\Allen(IIT-JEE Wing)\2020–21\Nurture\Che\Unit-08\Chemical Kinetics

211
JEE-Chemistr y

SINGLE CHOICE CORRECT QUESTIONS


1. In a reaction involving the synthesis of ammonia by Haber’s process,
N2 + 3H2 2NH3, the rate of reaction was measured as = 2.5 ×10–4 mol L–1 s–1. The rate of change of
conc. of H2 will be
(A) 1.25 × 10–4 mol L–1 s–1 (B) 2.50 × 10–4 mol L–1 s–1
–4 –1 –1
(C) 7.5 × 10 mol L s (D) 5.0 × 10–4 mol L–1 s–1

2. In the formation of sulphur trioxide by contact process, 2SO2 + O2 2SO3, the rate of reaction was
d[O2 ]
measured as = 2.5 × 10-4 mol lit-1 sec-1. the rate of formation of SO3 will be-
dt
(A) –1.25 × 10-4 mol L–2 sec-1 (B) 50 × 10-4 mol L–1 sec–1
(C) –3.75 × 10-4 mol L–1 sec-1 (D) 5.00 × 10-4 mol L–1 sec–1

d[C]
3. Reaction A + 2B + C product, follows the rate law : = k [A]2
dt
False statement regarding the above reaction is :
(A) On doubling the conc. of B and C the rate of the reaction remains unaffected
(B) Reducing the conc. of A to half, the rate becomes one-fourth
(C) Half life period of the reaction depends upon the conc. of B
(D) Half life period of the reaction is inversely proportional to the first power conc. of A

4. In a first order reaction, the initial conc. of the reactant was M/10. After 8 minutes 20 seconds the conc.
becomes M/100. What is the rate constant ?
(A) 5 × 10–3 second–1 (B) 2.303 × 10–5 second–1
–4 –1
(C) 2.303 × 10 second (D) 4.606 × 10–3 second–1

5. The rate law for the reaction : Ester + H+ Acid + Alcohol is


v = k [ester] [H3O+]0
What would be the new rate if
(a) Conc. of ester is doubled (b) Conc. of H+ is doubled
(A) (a) v (b) 2v (B) (a) 2v (b) v (C) (a) 2v (b) 2v (D) None of these

6. Dinitrogen pentaoxide decomposes as 2N 2O5 4NO2 + O2. the rate can be given in three ways
d[N2O5 ] d[NO2 ] d[O2 ]
= k1 [N2O5], = k2 [N2O5], = k3 [N2O5]
dt dt dt
The relation between the rate constants k1, k2 and k3 is :
(A) k2 = 2k1 and k3 = 1/2 k1 (B) k1 = 2k2 and k3 = 2k1
(C) k1 = k2 = k3 (D) k1 = 2k2 = 3k3

k1
JPR\COMP.251\D\Allen(IIT-JEE Wing)\2020–21\Nurture\Che\Unit-08\Chemical Kinetics

Y
d[x]
7. For a reaction X , is equal to
t 0 a dt
t t (a x) k2
Z

(A) k1 (a – x) – k2 (a – x) (B) k2 (a – x) – k1 (a – x)
(C) k1 (a – x) + k2 (a – x) (D) –k1 (a – x) – k2 (a – x)

8. The incorrect statement is-


(A) All the collisions between reactant molecules do not lead to a chemical change
(B) A zero order reaction proceeds at a constant rate independent of concentration or time
(C) Fast reactions have low activation energies
(D) In a first order reaction, the reaction ideally takes finite time to be completed
212
Chemical Kinetics

9. The reaction : 2A 2B + C occurs by the mechanism :


k1 k2
A B + X (slow) ....... (1) A+X B + C(fast) ....... (2)
Which of the following step is incorrect ?
(A) Molecularity of the reaction is one (B) The order of the reaction with respect to A is two
(C) The rate law is : v = k1 [A] (D) k2 > k1

10. The mechanism of the reaction : A + 2B + C D is


(step-1) (fast) equilibrium A + B X
(step-2) (slow) X + C Y
(step-3) (fast) Y + B D
Which rate law is correct
(A) v = k[C] (B) v = k [A] [B]2 [C]
(C) v = k [A] [B] [C] (D) v = k [D]

11. Mechanism of the reaction 2NO + Cl2 2NOCl may be written as


K k
2NO (NO)2 ............. (fast) (NO)2 + Cl2 2NOCl ............ (slow)
Rate equation would be
(A) kK[(NO)2] [Cl2] (B) kK[NO]2 [Cl2] (C) kK[Cl2] (D) kK[NO] [Cl2]

12. In a certain reaction 10% of the reactant decomposes in one hour, 20% in two hours, 30% in three hours and so
on. Dimension of the velocity constant (rate constant) are-
(A) Hour –1 (B) Mol litre–1hour –1
–1 –1
(C) Litre mol sec (D) Mol sec–1

13. For a reaction A + 2B C + D, the following data were obtained


Expt. Initial concentration Initial Rate of formation of D
(moles litre–1) (moles litre–1 min–1)
[A] [B]
1. 0.1 0.1 6.0 × 10–3
2. 0.3 0.2 7.2 × 10–2
3. 0.3 0.4 2.88 × 10–1
4. 0.4 0.1 2.4 × 10–2
The correct rate law expression will be :
(A) Rate = k [A] [B] (B) Rate = k [A] [B]2 (C) Rate = k [A]2 [B]2 (D) Rate = k [A]2 [B]

14. In acidic medium the rate of reaction between (BrO 3) – and Br– ion is given by the expression
d[BrO3 ]
= k[BrO3–] [Br–] [H+]2 it means
dt
(A) Rate constant of overall reaction is 4 sec –1
(B) Rate of reaction is independent of the conc. of acid
(C) The change in pH of the solution will not affect the rate
(D) Doubling the conc. of H+ ions will increase the reaction rate by 4 times
JPR\COMP.251\D\Allen(IIT-JEE Wing)\2020–21\Nurture\Che\Unit-08\Chemical Kinetics

d[D]
15. The rate law for the reaction : 2 C + D A + E is – = k[C]2 [D]
dt
If C is present in large excess, the order of the reaction will be -
(A) Zero (B) First (C) Second (D) Third

dx
16. For a second order reaction = k(a – x)2. Its half life period is :
dt
1 0.693 a 0.693
(A) (B) (C) (D)
a. k k k ak

213
JEE-Chemistr y
17. The dissociation of nitrogen pentoxide is a first order reaction. In first 24 minutes 75% of nitrogen pentaoxide is
dissociated. What amount of nitrogen pentaoxide will be left behind after one hour of start of the
reaction ?
(A) Approximately 1% (B) Approximately 2% (C) Approximately 3% (D) None

18. In the Wilhelmey equation of a first order reaction Ct = C0e–kt if the initial concentration C0 is increased m times
then
(A) The value of k will increase m times (B) The value of k will decrease m times
(C) The value of k will remains unchanged (D) None of these

19. The half-life period for a reaction at initial concentration of 0.5 and 1.0 moles litre –1 are 200 sec and 100 sec
respectively. The order of the reaction is
(A) 0 (B) 1 (C) 2 (D) 3

1
20. In Arrehenius equation if a graph is plotted between log k and , the slope of the curve will be :
T
Ea Ea E Ea
(A) – (B) – (C) a (D)
R 2.303 R R 2.303 R

21. A reaction takes place in three steps. The rate constants are k1, k2 and k3. The over all rate constant
k1k 3
k= . If (energy of activation) E1, E2 and E3 are 60, 30 and 10 kJ. The overall energy of activation is :
k2
(A) 40 (B) 30 (C) 400 (D) 60

22. The rate constant for the forward reaction A (g) 2B (g) is 1.5 × 10–3 s–1 at 100 K. If 10–5 moles of A and
100 moles of B are present in a 10 litre vessel at equilibrium then rate constant for the backward reaction at this
temperature is :
(A) 1.50 × 104 L mol–1 s–1 (B) 1.5 × 1011 L mol–1 s–1
10 –1 –1
(C) 1.5 × 10 L mol s (D) 1.5 × 10–11 L mol–1 s–1
1 1
2 2
23. Reaction A + B C + D follow's following rate law rate = k = [A] [B] . Starting with initial conc. of one
mole of A and B each, what is the time taken for amount of A of become 0.25 mole. Given
k = 2.31 × 10–3 sec–1.
(A) 300 sec. (B) 600 sec. (C) 900 sec. (D) none of these

24. The rate constant, the activation energy and the Arrhenius parameter of a chemical reaction at 25°C are
3.0× 10–4 s–1, 104.4 kJ mol–1 and 6.0 × 1014 s–1 respectively. The value of the rate constant at T is :
18 –1 14 –1 30 –1
(A) 2.0 × 10 s (B) 6.0 × 10 s (C) infinity (D) 3.6 × 10 s

25. The reactions of higher order are rare because :


(A) many body collisions involve very high activation energy
JPR\COMP.251\D\Allen(IIT-JEE Wing)\2020–21\Nurture\Che\Unit-08\Chemical Kinetics

(B) many body collisions have a very low probability


(C) many body collisions are not energetically favoured.
(D) many body collisions can take place only in the gaseous phase

214
Chemical Kinetics

SECTION - 1 : MULTIPLE CHOICE CORRECT QUESTIONS


1. Select the correct statements :
(A) The molecularity of an elementary reaction indicates how many reactant molecules take part in the step.
(B) The rate law of an elementary reaction can be predicted by simply seeing the stoichiometry of reaction.
(C) The slowest elementary step in sequence of the reactions governs the overall rate of formation of product.
(D) A rate law is often derived from a proposed mechanism by imposing the steady state approximation or
assuming that there is a pre-equilibrium.

d[A]
2. For the reaction A B, the rate law expression is = k [A]1/2. If initial concentration of [A] is [A] 0, then
dt
2 1/ 2
(A) The integerated rate expression is k = (A 0 A1 / 2 )
t

(B) The graph of A Vs t will be

K
(C) The half life period t 1 / 2 =
2[A ]10/ 2

[A]0
(D) The time taken for 75% completion of reaction t 3 / 4 =
k

3. Select incorrect statement(s):


(A) Unit of pre-exponential factor (A) for second order reaction is mol L–1 s–1.
(B) A zero order reaction must be a complex reaction.
(C) Molecularity is defined only for RDS in a complex reaction.
(D) Decay constant ( ) of radioactive substance is affected by temperature.

4. For the reaction CH4 + Br2 CH3Br + HBr, the experimental data require the following rate equation:
d k1[CH4 ][Br2 ]
[CH3Br] =
dt 1 k 2 [HBr ] /[Br2 ]
Which of the following is/are true regarding this ?
(A) The reaction is a single step reaction
(B) The reaction is 2nd order in the initial stages {[HBr] 0}
(C) The reaction is 2nd order in the final stages {[Br 2] 0}
(D) The molecularity of the reaction is two.
JPR\COMP.251\D\Allen(IIT-JEE Wing)\2020–21\Nurture\Che\Unit-08\Chemical Kinetics

5. Which of the following statement is incorrect ?


(A) The order of reaction is the sum of powers of all the concentration terms in the rate equation.
(B) The order of reaction with respect to one reactant is the ratio of the change of logarithm of the rate of the
reaction to the change in the logarithm of the concentration of the particular reactant, keeping the concentrations
of all other reactants constant.
(C) Orders of reactions can not be fractional.
(D) The order of a reaction can only be determined from the stoichiometric equation for the reaction.

215
JEE-Chemistr y

E1 E2
6. In a consecutive reaction system A B C when E1 is much greater than E2, the yield of B
increase with
(A) increase in temperature (B) decreases in temperature
(C) increase in initial concentration of A (D) decrease in initial concentration of A

7. Consider the following case of COMPETING 1ST ORDER REACTIONS


k1 C
A
k2 D
After the start of the reaction at t = 0 with only A, the [C] is equal to the [D] at all times. The time in which all
three concentrations will be equal is given by
1 1 1 1
(A) t = n3 (B) t = n3 (C) t = n2 (D) t = n2
2k 1 2k 2 3k 1 3k 2

8. For a reaction the Rate Law is


Rate = k[A]a[C]b
Where ‘C’ act as catalyst. At an initial concentration [A] = 0.1 M, [C] = 0.01 M, t1/3 and t5/9 was found to be
30 min and 75 min respectively. But t 1/3 was found to be 7.5 min if initial concentration are [A] = 0.2M and
[C]=0.02 M.
Select the correct option(s). [Given tx is time required for x fraction of reactant the react]
(A) a = 2
(B) b = 2
(C) When initial concentration are [A] = 0.2 M, [C] = 0.02 M, t 5/9 is 18.75 min.
(D) When initial concentration are [A] = 0.2 M, [C] = 0.02 M, t 5/9 is 22.5 min.

9. The polarimeter readings in a experiment to measure the rate of inversion of cane sugar (1 st order reaction) were
as follows:
Time (min.) 0 30
Rotation due to solution (degree) 30 20 –15
Identify the true statements:
[Given : log 2 = 03, log 3 = 0.48, log 7 = 0.84, (log e 10) = 2.3
(A) The half life of the reaction is 75 min
(B) The solution is optically inactive at 120 min
(C) The half life of the reaction is 60 min
(D) The solution is optically inactive at 90 min.

10. For a certain reaction A P, the half life for different initial concentration of A is mentioned below.
[A0] 0.1 0.025
t½ (s) 100 50
Which of the following option(s) is/are incorrect.
(A) The order is 1 (B) The order is 0.5
(C) For [A0] = 1M, t½ = 100 10 sec . (D) For [A0] = 1M t½ = 100 sec.
JPR\COMP.251\D\Allen(IIT-JEE Wing)\2020–21\Nurture\Che\Unit-08\Chemical Kinetics

SECTION - 2 : COMPREHENSION BASED QUESTIONS


(SINGLE CHOICE CORRECT QUESTION)
Comprehension-1

216
Chemical Kinetics
11. For the (Set-1) :
(A) if T1 > T2, k1 > k2 always (B) if T1 > T2, k1 > k2 (for exothermic reaction)
(C) if T1 > T2, k1 < k2 (for endothermic reaction) (D) Ea1 Ea2

12. For the (Set-1) :


(A) Ea1 > Ea2 if T1 > T2 (B) Ea1 < Ea2 if T1 > T2
(C) Ea1 = Ea2 (D) Ea1 = 0.5 Ea2

13. Comparing set-I and II :


(A) k4 > k3 & k2 > k1 , if T2 > T1 (endothermic) (B) k4 < k3 & k2 > k1 , if T2 < T1 (endothermic)
(C) k4 > k3 & k2 > k1 , if T2 < T1 (exothermic) (D) k4 < k3 & k2 < k1, if T2 > T1 (exothermic)

Comprehension-2
A reaction is said to be first order if it's rate is proportional to the concentration of reactant. Let us consider a
reaction
A(g) B(g) + C(g)
At t = 0 a 0 0
At time t a–x x x
dx
The rate of reaction is given by the expression = k(a – x) and integrated rate equation for a given reaction
dt
1 a
is represented as k = ln where a = initial concentration and (a – x) = concentration of A after
t a x
time t.

14. Thermal decomposition of compound X is a first order reaction. If 75% of X is decomposed in 100 min. How
long will it take for 90% of the compound to decompose?
Given : log 2 = 0.30
(A) 190 min (B) 176.66 min (C) 166.66 min (D) 156.66 min

15. Consider a reaction A(g) 3B(g) + 2C(g) with rate constant 1.386 × 10–2 min–1. Starting with
2 moles of A in 12.5 litre vessel initially, if reaction is allowed to takes place at constant pressure & at 298K then
find the concentration of B after 100 min.
(A) 0.04 M (B) 0.36 M (C) 0.09 M (D) None of these

Comprehension-3
Competing First-Order Reactions
Frequently a species can react in different ways to give a variety of products. For example, toluene can be
nitrated at the ortho, meta, or para positions, We shall consider the simplest case, that of two competing
irreversible first-order reactions :
k1 k2 D
A C and A
where the stoichiometric coefficients are taken as unity for simplicity. The rate law is
d [A ]
= – k1[A] – k2[A] = – (k1 + k2) [A] [A] = [A]0 e ( k 1 k 2 )t .
dt

d [C]
For C, we have = k1[A] = k1[A]0 e ( k 1 k 2 )t . Multiplication by dt and integration from time 0
JPR\COMP.251\D\Allen(IIT-JEE Wing)\2020–21\Nurture\Che\Unit-08\Chemical Kinetics

dt
k 1[ A ] 0 ( k1 k 2 ) t
(where [C]0 = 0) to an arbitary time t gives [C] = (1 e )
k1 k 2
d [D] k [A]
Similarly, integration of gives [D] = 2 0 (1 – e (k 1 k 2 )t )
dt k1 k 2
The sum of the rate constants k1 + k2 appears in the exponentials for both [C] and [D].
[C] k1
At any time we also have =
[D] k2

217
JEE-Chemistr y

16. A starting initially with only A Which of the following is correct at time t

(A) [A]0 = [A]t +[B]t + [C]t (B) [A]0 = [A]t + 2 [B]t + 3 [C]t
[B] t [C] t 2
(C) [A]0 = [A]t + + (D) [A]0 = [A]t +[B]t + [C]t
2 3 3

[ X] t
17. X starting with only 'X', ratio
[ Y ] t [ Z] t

1
(A) Independent of time (B)
(ekt 1)
(C) Depends upon initial concentration of X (D) [A]0 (ekt –1)

18. For A starting with pure A ratio of rate of production of B to C is

(A) Independent of time (B) Independent of temperature


(C) Depends upon initial concentration of A (D) Independent of mechanism of reaction

JPR\COMP.251\D\Allen(IIT-JEE Wing)\2020–21\Nurture\Che\Unit-08\Chemical Kinetics

218
Chemical Kinetics

SECTION - 1 : NUMERICAL ANSWER BASED QUESTIONS


1. A first order reaction has a rate constant 1.5 × 10–3 sec–1. How long (in sec.) will 5.0 g of this reactant take
to reduce to 1.25 g.

2. The reaction SO2Cl2 (g) SO2 (g) + Cl2 (g) is a first order gas reaction with k = 2.2 × 10 –5 sec–1 at 320°C.
What % of SO2Cl2 is decomposed on heating this gas for 90 min.

3. Two substance A (t1/2 = 5 mins) and B (t1/2 = 15 mins) follow first order kinetics are taken in such a way that
initially [A] = 4[B]. Calculate the time (in minute) after which the concentration of both the substance will be
equal.

4. Calculate the age (in years) of a vegetarian beverage whose tritium content is only 15% of the level in living
plants. Given t1/2 for 1H3 = 12.3 years.

5. Decomposition of both A2(g) and B3 (g) follows 1st order kinetic as :


14000

K1(hr–1) = 102 e
K1
A2(g) 2A(g) RT

20000

B3(g) K2
3B(g) K2(hr–1) = 103 e RT
If one mole of each A2(g) and B3(g) are taken in a 10 L evacuated flask and heated to some temperature so that
they start decomposing at the same rate, determine total pressure (in atm) in the flask after 1.0 hr.

B
6. The reaction A proceeds in 1st order parallel channels A . Suppose the half life values for the two branches
C
are 60 minutes and 90 minutes, what is the overall half-life (in minute) value?

7. H2 gas is adsorbed on the metal surface like tungsten. The order of this process is

8. In a first order reaction, the concentration of the reactant, decreases from 0.8 M to 0.4 M in 15 minutes.
The time taken (in minutes) for the concentration to change from 0.1 M to 0.025 M is

9. The half-life of a radio isotope is four hours. If the initial mass of the isotope was 200 g, the mass (in gm)
remaining after 24 hours undecayed is :

10. A certain organic compound A decomposes by two parallel first order mechanism
k1 B
A
If k1 : k2 = 1 : 9 and k1 = 1.3 × 10–5 s–1
k2
C
Calculate the concentration ratio of C to A, if experiment is started with only A and allowed to run for one hour.
JPR\COMP.251\D\Allen(IIT-JEE Wing)\2020–21\Nurture\Che\Unit-08\Chemical Kinetics

219
JEE-Chemistr y
SECTION - 2 : MATRIX - MATCH QUESTIONS
11. For the reaction of type A(g) 2B(g)
Column-I contains four entries and column-II contains four entries. Entry of column-I are to be matched with
ONLY ONE ENTRY of column-II.

Column - I Column - II

d[ B] d[ A]
(A) vs for first order (p)
dt dt

(B) [A] vs t for first order (q)

(C) [B] vs t for first order (r)

(D) [A] vs t for zero order (s)

12. Match the following :


Column-I Column-II
(A) If the activation energy is 65 kJ then how much time (p) 2
faster a reaction proceed at 25°C than at 0°C

(B) Rate constant of a first - order reaction is 0.0693 min–1. (q) Zero
If we start with 20 mol L–1, it is reduced to 2.5 mol L–1
in how many minutes

(C) Half - lives of first - order and zero order reactions (r) 11
are same. Ratio of rates at the start of reaction is
how many times of 0.693 Assume initial concentration to
be same for the both.

(D) The half-life periods are given , (s) 30


[A]0 (M) 0.0677 0.136 0.272
t1/2 (sec) 240 480 960
order of the reaction is
JPR\COMP.251\D\Allen(IIT-JEE Wing)\2020–21\Nurture\Che\Unit-08\Chemical Kinetics

13. The polarimeter readings in an experiment to measure the rate of inversion of cane suger (1st order reaction)
were as follows
time (min) : 0 30
angle (degree) : 30 20 – 15
Then match the following. (Use : log 35 = 1.54, log 3 = 0.48, log 2 = 0.3)
Column - I Column - II
(A) The half life of the reaction (p) 120 min.

(B) The solution is optically inactive at (q) 7.5°

(C) The equimolar mixture of the products (r) 75.2 min.

(D) The angle at half time (s) laevorotatory

220
Chemical Kinetics

1
1. For a reaction A 2B , rate of disappearance of ‘A’ is related to the rate of appearance of ‘B’ by the
2
expression [AIEEE-2008]
d A 1d B d A 1d B d A d B d A d B
(1) – = (2) – = (3) – = (4) – =4
dt 2 dt dt 4 dt dt dt dt dt

2. The half life period of a first order chemical reaction is 6.93 minutes. The time required for the completion
of 99% of the chemical reaction will be (log 2 = 0.301) :- [AIEEE-2009]
(1) 46.06 minutes (2) 460.6 minutes (3) 230.3 minutes (4) 23.03 minutes

3. The time for half life period of a certain reaction A Products is 1 hour. When the initial concentration fo
–1,
the reactant 'A' is 2.0 mol L How much time does it take for its concentration to come from 0.50 to 0.25
mol L–1 if it is a zero order reaction ? [AIEEE-2010]
(1) 1 h (2) 4 h (3) 0.5 h (4) 0.25 h

4. The rate of a chemical reaction doubles for every 10°C rise of temperature. If the temperature is raised by 50°C,
the rate of the reaction increases by about :- [AIEEE-2011]
(1) 32 times (2) 64 times (3) 10 times (4) 24 times

5. A reactant (A) forms two products :


k k1
A 1
B, Activation Energy Ea1 A C, Activation Energy Ea2
If Ea2 = 2 Ea1, then k1 and k2 are related as :- [AIEEE-2011]
Ea1 / RT
(1) k1 = 2k 2 e Ea 2 / RT
(2) k1 = k 2eEa1 / RT (3) k2 = k1e Ea2 / RT (4) k1 = A k 2 e

6. The activation energy for a reaction which doubles the rate when the temperature is raised from 298 K to 308
K is :- [AIEEE ONLINE - 2012]
(1) 29.5 kJ mol–1 (2) 39.2 kJ mol–1 (3) 52.9 kJ mol–1 (4) 59.2 kJ mol–1

7. In a chemical reaction A is converted into B. The rates of reaction, starting with initial concentration of A as
2 × 10–3 M and 1 × 10–3 M, are equal to 2.40 × 10–4 Ms–1 and 0.60 × 10–4 Ms–1 respectively. The order
of reaction with respect to reactant A will be :- [AIEEE ONLINE - 2012]
(1) 1.5 (2) 2 (3) 0 (4) 1

8. For a first order reaction, (A) products, the concentration of A changes from 0.1 M to 0.025M in 40 minutes.
The rate of reaction when the concentration of A is 0.01 M is :- [AIEEE-2012]
(1) 1.73 × 10–4 M/min (2) 1.73 × 10–5 M/min (3) 3.47 × 10–4 M/min (4) 3.47 × 10–5 M/min

9. A radioactive isotope having a half - life period of 3 days was received after 12 days. If 3g of the isotope is
left in the container, what would be the initial mass of the isotope ? [JEE MAINS - ONLINE - 2013]
JPR\COMP.251\D\Allen(IIT-JEE Wing)\2020–21\Nurture\Che\Unit-08\Chemical Kinetics

(1) 36g (2) 48g (3) 24g (4) 12g

10. The half-life period of a first order reaction is 15 minutes. The amount of substance left after one hour will be:
[JEE MAINS - ONLINE - 2014]
1 1
(1) of the original amount (2) of the original amount
4 16
1 1
(3) of the original amount (4) of the original amount
32 8

221
JEE-Chemistr y
11. For the reaction
3A + 2B C + D the differential rate law can be written as [JEE MAINS - ONLINE - 2014]
1 d[A] d[C] 1 d[A] d[C]
(1) k[A]n [B]m (2) k[A]n [B]m
3 dt dt 3 dt dt
1 d[A] d[C] d[A] d[C]
(3) k[A]n [B]m (4) k[A]n [B]m
3 dt dt dt dt

12. For the non-stoichiometry reaction 2A + B C + D, the following kinetic data were obtained in three separate
experiments, all at 298 K. [JEE Mains-2014]

Initial Initial Initital rate of


Concentration Concentration formationofC
(A) (B) (mol L S )
3
0.1M 0.1M 1.2 10
3
0.1M 0.2M 1.2 10
3
0.2M 0.1M 2.4 10

The rate law for the formation of C is:


dc dc dc dc
(1) k[A][B] (2) k[A]2 [B] (3) k[A][B]2 (4) k[A]
dt dt dt dt

13. A + 2B C, the rate equation for this reaction is given as Rate = k[A][B].[JEE MAINS - ONLINE - 2015]
If the concentration of A is kept the same but that of B is doubled what will happen to the rate itself?
(1) quadrupled (2) doubled (3) halved (4) the same

14. The rate law for the reaction below is given by the expression k[A] [B]
A+B Product [JEE MAINS - ONLINE - 2016]
If the concentration of B is increased from 0.1 to 0.3 mole, keeping the value of A at 0.1 mole, the rate constant
will be :
(1) 3k (2) 9k (3) k (4) k/3

15. For the reaction, 2A + B products, when the concentrations of A and B both wrere doubled, the rate of
the reaction increased from 0.3 mol L–1s–1 to 2.4 mol L–l s–1. When the concentration of A alone is doubled,
the rate increased from 0.3 mol L–1s–1 to0.6 mol L–1s–1
Which one of the following statements is correct ? [JEE MAINS - ONLINE - 2019]
(1) Order of the reaction with respect to B is 2 (2) Order of the reaction with respect to A is 2
(3) Total order of the reaction is 4 (4) Order of the reaction with respect to B is 1

16. Consider the following reversible chemical reactions :


K1
A2(g) + Br2(g) 2AB(g) .....(1) [JEE MAINS - ONLINE - 2019]
JPR\COMP.251\D\Allen(IIT-JEE Wing)\2020–21\Nurture\Che\Unit-08\Chemical Kinetics

K2
6AB(g) 3A2(g) + 3B2(g) .....(2)
The relation between K1 and K2 is :
1
(1) K2 = K13 (2) K2 = K1–3 (3) K1K2 = 3 (4) K1K2 =
3

222
Chemical Kinetics
17. The following results were obtained during kinetic studies of the reaction :
2A + B Products [JEE MAINS - ONLINE - 2019]
Experment [A] [B] Initial Rate of reaction
-1 -1
(in mol L ) (in mol L ) (in mol L–1 min–1)

(I) 0.10 0.20 6.93 × 10–3

(II) 0.10 0.25 6.93 × 10–3

(III) 0.20 0.30 1.386 × 10–2

The time (in minutes) required to consume half of A is :


(1) 10 (2) 5 (3) 100 (4) 1

18. For an elementary chemical reaction,


k1 d[A]
A2 2A, the expression for is : [JEE MAINS - ONLINE - 2019]
k –1
dt
(1) 2k1[A2]–k–1[A]2 (2) k1[A2]–k–1[A]2 (3) 2k1[A2]–2k–1[A]2 (4) k1[A2]+k–1[A]2

19. Consider the given plots for a reaction obeying Arrhenius equation (0°C < T < 300°C) : (k and Ea are rate
constant and activation energy, respectively) [JEE MAINS - ONLINE - 2019]

k k

Ea T(°C)
I II
Choose the correct option :
(1) Both I and II are wrong (2) I is wrong but II is right
(3) Both I and II are correct (4) I is right but II is wrong

20. The reaction 2X B is a zeroth order reaction. If the initial concentration of X is 0.2 M, the half-life is 6 h.
When the initial concentration of X is 0.5 M, the time required to reach its final concentration of 0.2 M will
be :- [JEE MAINS - ONLINE - 2019]
(1) 18.0 h (2) 7.2 h (3) 9.0 h (4) 12.0 h

1
21. If a reaction follows the Arrhenius equation, the plot lnk vs gives straight line with a gradient (–y) unit.
(RT)
The energy required to activate the reactant is : [JEE MAINS - ONLINE - 2019]
(1) y unit (2) –y unit (3) yR unit (4) y/R unit

22. For a reaction, consider the plot of ln k versus 1/T given in the figure. If the rate constant of this reaction at
400 K is 10–5 s–1, then the rate constant at 500 K is : [JEE MAINS - ONLINE - 2019]
JPR\COMP.251\D\Allen(IIT-JEE Wing)\2020–21\Nurture\Che\Unit-08\Chemical Kinetics

ln k slope = –4606K

1/T
–4 –1 –4 –1
(1) 2×10 s (2) 10 s (3) 10–6 s–1 (4) 4×10–4 s–1

23. Decomposition of X exhibits a rate constant of 0.05 g/year. How many years are required for the decomposition
of 5 g of X into 2.5 g ? [JEE MAINS - ONLINE - 2019]
(1) 50 (2) 25 (3) 20 (4) 40

223
JEE-Chemistr y
24. For the reaction 2A +B C, the values of initial rate at different reactant concentrations are given in the table
below. The rate law for the reaction is : [JEE MAINS - ONLINE - 2019]

[A] (mol L–1) [B] (mol L–1) Initial Rate


(mol L–1s–1)
0.05 0.05 0.045
0.10 0.05 0.090
0.20 0.10 0.72
(1) Rate = k [A][B] (2) Rate = k [A]2[B]2 (3) Rate = k [A][B]2 (4) Rate = k [A]2[B]

k1 k2
25. For a reaction scheme A B C , if the rate of formation of B is set to be zero then the concentration
of B is given by : [JEE MAINS - ONLINE - 2019]

k1
(1) [A] (2) (k1 + k2) [A] (3) k1k2[A] (4) (k1 – k2) [A]
k2

26. The given plots represent the variation of the concentration of a reactant R with time for two different reactions
(i) and (ii). The respective orders of the reactions are : [JEE MAINS - ONLINE - 2019]

(1) 1,0 (2) 1,1 (3) 0,1 (4) 0,2

27. In the following reaction; xA yB


d[A] d[B]
log10 log10 0.3010 [JEE MAINS - ONLINE - 2019]
dt dt
'A' and 'B' respectively can be :
(1) n-Butane and Iso-butane (2) C2H4 and C4H8
(3) N2O4 and NO2 (4) C2H2 and C6H6

JPR\COMP.251\D\Allen(IIT-JEE Wing)\2020–21\Nurture\Che\Unit-08\Chemical Kinetics

224
Chemical Kinetics

SECTION - 1 : SINGLE CHOICE CORRECT QUESTIONS


1. Consider the reaction:
Cl2(aq) + H2S(aq) S(s) + 2H+(aq) + 2Cl–(aq)
The rate equation for this reaction is
rate = k[Cl2][H2S]
Which of these mechanisms is/are consistent with this rate equation ? [AIEEE-2010]
a. Cl2 + H2S H+ + Cl– + Cl+ + HS– (slow)
Cl+ + HS– H+ + Cl– + S(fast)
b. H2S H + HS– (fast equilibrium)
+

Cl2 + HS– 2Cl– + H+ + S(slow)


(1) a only (2) b only (3) Both a and b (4) Neither a nor b

2. For a reaction A Products, a plot of log t½ versus log a0 is shown in figure. If the initial concentration of A is
represented by a0, the order of the reaction is :- [AIEEE ONLINE - 2012]
(1) Two
(2) Zero log t½ 45º
(3) One
(4) Three
log a0

3. The instantaneous rate of disappearance of MnO4 ion in the following reaction is 4.56 × 10 –3Ms–1

2MnO4– + 10 I– + 16 H+ 2 Mn2+ + 5I2 + 8H2O [JEE MAINS - ONLINE - 2013]


The rate of appearance I2 is :
(1) 1.14 × 10–2 Ms–1 (2) 5.7× 10-3 Ms–1 (3) 4.56 × 10–4 Ms–1 (4) 1.14 × 10–3 Ms–1

4. The rate constant of a zero order reaction is 2.0 × 10 –2 mol L–1 s–1. If the concentration of the reactant after 25
seconds is 0.5 M. What is the initial concentration ? [JEE MAINS - ONLINE - 2013]
(1) 0.5 M (2) 12.5 M (3) l.0 M (4) 1.25 M

5. The rate of reaction doubles when its temperature changes form 300 K to 310 K. Activation energy of such a
reaction will be : (R = 8.314 JK–1 mol–1 and log 2 = 0.301) [JEE Mains-2013]
(1) 53.6 kJ mol–1 (2) 48.6 kJ mol–1 (3) 58.5 kJ mol–1 (4) 60.5 kJ mol–1

d[N 2O5 ]
6. For the reaction, 2N2O5 4NO2 + O2, the rate equation can be expressed in two ways k[N 2O5 ]
dt
d[NO 2 ]
and k [N 2O5 ] [JEE MAINS - ONLINE - 2014]
dt
k and k are related as
(1) k = k (2) k = 4k (3) 2k = k (4) k = 2k

The rate coefficient (k) for a particular reactions is 1.3 x 10–4 M–1 s–1 at 100ºC, and 1.3 × 10– 3 M–1 s–1 at
JPR\COMP.251\D\Allen(IIT-JEE Wing)\2020–21\Nurture\Che\Unit-08\Chemical Kinetics

7.
150°C, What is the energy of activation (E A) (in kJ) for this reaction ?
(R = molar gas constant = 8.314 JK–1 mol–1) [JEE MAINS - ONLINE - 2014]
(1) 132 (2) 60 (3) 99 (4) 16

8. The reaction
2N2O5(g) 4NO2(g) + O2(g) [JEE MAINS - ONLINE - 2015]
follows first order kinetics. The pressure of a vessel containing only N 2O5 was found to increase from 50 mm Hg
to 87.5 mm Hg in 30 min. The pressure exerted by the gases after 60 min. will be (Assume temperature remains
constant) :
(1) 106.25 nm Hg (2) 116.25 nm Hg (3) 125 mm Hg (4) 150 mm Hg

225
JEE-Chemistr y
9. Decomposition of H2O2 follows a first order reaction. In fifty minutes the concentration of H2O2 decreases from
0.5 to 0.125 M in one such decomposition. When the concentration of H2O2 reaches 0.05 M, the rate of
formation of O2 will be [JEE Mains-2016]
(1) 6.93 × 10–2 mol min–1 (2) 6.93 × 10–4 mol min–1
(3) 2.66 L min–1 at STP (4) 1.34 × 10–2 mol min–1

10. The rate of a reaction A doubles on increasing the temperature from 300 to 310 K. By how much, the temperature
of reaction B should be increased from 300 K so that rate doubles if activation energy of the reeaction B is twice
to that of reaction A : [JEE MAINS - ONLINE - 2017]
(1) 2.45 K (2) 4.92 K (3) 9.84 K (4) 19.67 K

11. The rate of a reaction quadruples when the temperature changes from 300 to 310 K. The activation energy of
this reaction is: [JEE MAINS - ONLINE - 2017]
(Assume activation energy and pre-exponential factor are independent of temperature; n 2 = 0.693;
R = 8.314 J mol–1 K–1)
(1) 107.2 kJ mol–1 (2) 53.6 kJ mol–1 (3) 214.4 kJ mol–1 (4) 26.8 kJ mol–1

12. Two reactions R1 and R2 have identical pre-exponential factors. Activation energy of R1 exceeds that R2 by 10 kJ
mol–1. If k1 and k2 are rate constants for reactions R1 and R2 respectively at 300 K, then In(K2/k1) is equal to :
(R = 8.314 J mol–1 K–1) [JEE Mains-2017]
(1) 4 (2) 8 (3) 12 (4) 6

13. At 518ºC, the rate of decomposition of a sample of gaseous acetaldehyde, initially at a pressure of 363 Torr, was
1.00 Torr s–1 when 5% had reacted and 0.5 Torr s–1 when 33% had reacted. The order of the reaction is
[JEE Mains-2018]
(1) 2 (2) 3 (3) 1 (4) 0

14. A bacterial infection in an internal wound grows as N'(t) = N 0 exp(t), where the time t is in hours. A dose of
antibiotic, taken orally, needs 1 hour to reach the wound. Once it reaches there, the bacterial population goes
dN N0
down as = –5N2. What will be the plot of vs. t after 1 hour ? [JEE MAINS - ONLINE - 2019]
dt N

N0 N0 N0 N0
N N N N
(1) (2) (3) (4)

t(h) t(h) t(h) t(h)

15. For the reaction of H2 with I2, the rate constant is 2.5×10–4dm3 mol–1 s–1 at 327°C and 1.0 dm3 mol–1 s–1 at
527°C. The activation energy for the reaction, in kJ mol –1 is: [JEE MAINS - ONLINE - 2019]
(R=8.314J K–1 mol–1)
(1) 72 (2) 166 (3) 150 (4) 59

16. NO2 required for a reaction is produced by the decomposition of N 2O5 in CCl4 as per the equation
2N2O5(g) 4NO2(g) + O2(g). [JEE MAINS - ONLINE - 2019]
JPR\COMP.251\D\Allen(IIT-JEE Wing)\2020–21\Nurture\Che\Unit-08\Chemical Kinetics

The initial concentration of N2O5 is 3.00 mol L–1


and it is 2.75 mol L–1 after 30 minutes. The rate of formation of NO2 is :
(1) 2.083 × 10–3 mol L–1 min–1 (2) 4.167 × 10–3 mol L–1 min–1
–3 –1 –1
(3) 8.333 × 10 mol L min (4) 1.667 × 10–2 mol L–1 min–1

17. For the following reactions [JEE MAINS - ONLINE - 2020]


500 K
A 700K
Product A catalyst
Product
it was found that Ea is decreased by 30 kJ/mol in the presence of catalyst.
If the rate remains unchanged, the activation energy for catalysed reaction is (Assume pre exponential factor is
same):
(1) 135 kJ/mol (2) 105 kJ/mol (3) 198 kJ/mol (4) 75 kJ/mol

226
Chemical Kinetics

SECTION - 2 : NUMERICAL ANSWER BASED TYPE QUESTIONS


1. During the nuclear explosion, one of the products is 90Sr with half life of 6.93 years. if 1 g of 90Sr was absorbed
in the bones of a newly born baby in place of Ca, how much time, in years , is required to reduce it by 90% if
it is not lost metabolically______ [JEE MAINS - ONLINE - 2020]

2. A sample of milk splits after 60 min. at 300 K and after 40 min. at 400 K when the population of lactobacillus
acidophilus in it doubles. The activa tion energy (in kJ/ mol) for this process is closest to__________.
2
(Given, R = 8.3 J mol–1 K–1, ln = 0.4, e–3 = 4.0) [JEE MAINS - ONLINE - 2020]
3
JPR\COMP.251\D\Allen(IIT-JEE Wing)\2020–21\Nurture\Che\Unit-08\Chemical Kinetics

227
JEE-Chemistr y

*1. Under the same reaction conditions, initial concentration of 1.386 mol dm–3 of a substance becomes half in 40

k1
seconds and 20 seconds through first order and zero order kinetics, respectively. Ratio of the rate constants
k0
for first order (k1) and zero order (k0) of the reactions is [JEE 2008]
(A) 0.5 mol–1 dm3 (B) 1.0 mol dm–3 (C) 1.5 mol dm–3 (D) 2.0 mol–1 dm3

2. Statement-1 : The plot of atomic number (y-axis) versus number of neutrons (x-axis) for stable nuclei shows a
curvature towards x-axis from the line of 45° slope
as the atomic number is increased.
and
Statement-2 : Proton-proton electrostatic repulsions begin to overcome attractive forces involving protons and
neutrons in heavier nuclides. [JEE 2008]
(A) Statement-1 is True, Statement-2 is True ; Statement-2 is a correct explanation for Statement-1
(B) Statement-1 is True, Statement-2 is True ; Statement-2 is NOT a correct explanation for Statement-1
(C) Statement-1 is True, Statement-2 is False
(D) Statement-1 is False, Statement-2 is True

238 214
*3. The total number of and particles emitted in the nuclear reaction 92 U 82 Pb is. [JEE 2009]

142
4. The number of neutrons emitted when 235
92 U undergoes controlled nuclear fission to 54 Xe and 90
38 Sr is -
[JEE 2010]

5. Plots showing the variation of the rate constant (k) with temperature (T) are given below. The plot that follows
Arrhenius equation is – [JEE 2010]

k k k k
(A) (B) (C) (D)

T T T T

*6. The concentration of R in the reaction R P was measured as a function of time and the following data
is obtained :

[R] (molar) 1.0 0.75 0.40 0.10

t(min.) 0.0 0.05 0.12 0.18


JPR\COMP.251\D\Allen(IIT-JEE Wing)\2020–21\Nurture\Che\Unit-08\Chemical Kinetics

The order of the reaction is. [JEE 2010]

7. For the first order reaction [JEE 2011]


2N2O5(g) 4NO2(g) + O2(g)
(A) the concentration of the reactant decreases exponentially with time
(B) the half-life of the reaction decreases with increasing temperature.
(C) the half-life of the reaction depends on the initial concentration of the reactant.
(D) the reaction proceeds to 99.6% completion in eight half-life duration.

228
Chemical Kinetics

*8. Bombardment of aluminium by -particle leads to its artificial disintegration in two ways,
(i) and (ii) as shown. Products X, Y and Z respectively are : [JEE 2011]
27 (ii) 30
13
Al 15 P + Y
(i)

30 30
14 Si + X 14Si + Z
(A) proton, neutron, positron (B) neutron, positron, proton
(C) proton, positron, neutron (D) positron, proton, neutron

*9. The periodic table consists of 18 groups. An isotope of copper, on bombardment with protons, undergoes a
nuclear reaction yielding element X as shown below. To which group , element X belongs in the periodic table?
[JEE 2012]
63 1
29 Cu 1 H 6 10 n 211 H X

10. An organic compound undergoes first-order decomposition . The time taken for its decomposition to 1/8 and

[t 1/8 ]
1/10 of its initial concentration are t1/8 and t1/10 respectively. What is the value of 10 ?
t 1/10
(Take log102 = 0.3) [JEE 2012]

*11. In the reaction,


P+Q R+S
the time taken for 75% reaction of P is twice the time taken for 50% reaction of P. The concentration of Q varies
with reaction time as shown in the figure. The overall order of the reaction is [JEE 2013]

[Q]0

[Q]

Time

(A) 2 (B) 3 (C) 0 (D) 1

12. For the elementary reaction M N, the rate of disappearance of M increases by a factor of 8 upon doubling
the concentration of M. The order of the reaction with respect to M is [JEE 2014]
(A) 4 (B) 3 (C) 2 (D) 1
JPR\COMP.251\D\Allen(IIT-JEE Wing)\2020–21\Nurture\Che\Unit-08\Chemical Kinetics

*13. A closed vessel with rigid walls contains 1 mol of 238 and 1 mol of air at 298 K. Considering complete decay
92 U

of 238 to 206 the ratio of the final pressure to the initial pressure of the system at 298 K is [JEE 2015]
92 U 82 Pb ,

*14. A plot of the number of neutrons (N) against the number of protons (P) of stable nuclei exhibits upwards
deviation from linearity for atomic numberk, Z > 20. For an unstable nucleus having N/P ratio less than 1, the
possible mode(s) of decay is (are) [JEE 2016]
(A) – -decay ( emission) (B) orbital or K-electron capture
(C) neutron emission (D) -decay (positron emission)

229
JEE-Chemistr y
*15. According to the Arrhenius equation, [JEE 2016]
(A) a high activation energy usually implies a fast reaction.
(B) rate constant increases with increase in temperature. This is due to a greater number of collisions whose
energy exceeds the activation energy.
(C) higher the magnitude of activation energy, stronger is the temperature dependence of the rate constant.
(D) the pre–exponential factor is a measure of the rate at which collisions occur, irrespective of their energy.

16. In a bimolecular reaction, the steric factor P was experimentally determined to be 4.5. The correct option(s)
among the following is (are) [JEE 2017]
(A) The value of frequency factor predicted by Arrhenius equation is higher than that determined experimentally
(B) The activation energy of the reaction is unaffected by the value of the steric factor
(C) Since P = 4.5, the reaction will not proceed unless an effective catalyst is used
(D) Experimentally determined value of frequency factor is higher than that predicted by Arrhenius equation

*17. Consider the following reversible reaction [JEE 2018]

A(g) B(g) AB(g)


The activation energy of the backward reaction exceeds that of the forward reaction by 2RT (in J mol –1). If the
pre–exponential factor of the forward reaction is 4 times that of the reverse reaction, the absolute value G
(in J mol–1) for the reaction at 300 K is :
(Given: n(2) = 0.7, RT = 2500 J mol–1 at 300 K and G is the Gibbs energy)

*18. For a first order reaction A(g) 2B(g) + C(g) at constant volume and 300 K, the total pressure at the beginning
(t = 0) and at time t are P0 and Pt, respectively. Initially, only A is present with concentration [A]0, and t1/3 is the
time required for the partial pressure of A to reach 1/3 rd of its initial value. The correct option(s) is (are)
(Assume that all these gases behave as ideal gases) [JEE 2018]
In(3P0 – Pt)

t1/3

(A) (B)

Time [A]0
Rate constant
In(P0 – Pt)

(C) (D)

Time [A]0

19. In the decay sequence :


–x1 234 –x2 –x3 234 –x4 234
JPR\COMP.251\D\Allen(IIT-JEE Wing)\2020–21\Nurture\Che\Unit-08\Chemical Kinetics

238 234
92 U 90 Th 91 Pa Z 90 Th
[JEE 2019]
x1, x2, x3 and x4 are particles/ radiation emitted by the respective isotopes. The correct option(s) is/are-
(A) Z is an isotope of uranium
(B) x2 is –
(C) x1 will deflect towards negatively charged plate
(D) x3 is -ray

230
Chemical Kinetics
20. Consider the kinetic data given in the following table for the reaction A + B + C Product.

Experiment [A] [B] [C] Rate of reaction


No. (mol dm–3) (mol dm–3) (mol dm–3) (mol dm–3 s–1)
1 0.2 0.1 0.1 6.0 × 10–5
2 0.2 0.2 0.1 6.0 × 10–5
3 0.2 0.1 0.2 1.2 × 10–4
4 0.3 0.1 0.1 9.0 × 10–5
The rate of the reaction for [A] = 0.15 mol dm –3, [B] = 0.25 mol dm–3 and [C] = 0.15 mol dm–3 is found to
be Y × 10–5 mol dm–3 s–1. The value of Y is –––––––– [JEE 2019]
JPR\COMP.251\D\Allen(IIT-JEE Wing)\2020–21\Nurture\Che\Unit-08\Chemical Kinetics

231
JEE-Chemistr y

ANSWERS
EXERCISE-1
Que. 1 2 3 4 5 6 7 8 9 10 11 12 13 14 15
Ans. C B C D B A C D B C B B B D B
Que. 16 17 18 19 20 21 22 23 24 25
Ans. A C C C B A D B B B

EXERCISE-2
MULTIPLE CORRECT CHOICE QUESTIONS
1. (ABCD) 2. (ABD) 3. (ACD) 4. (BC) 5. (CD)
6. (AC) 7. (AB) 8. (AC) 9. (AB) 10. (AD)

COMPREHENSION BASED QUESTIONS


11. (A) 12. (C) 13. (A) 14. (C) 15. (C)
16. (C) 17. (B) 18. (A)

EXERCISE-3
NUMERICAL ANSWER BASED QUESTIONS
1. (924) 2. (11.2) 3. (15) 4. (33.67) 5. (5.42)
6. (36) 7. (0) 8. (30) 9. (3.125) 10. (0.537)

MATRIX MATCH QUESTIONS


11. (A) - (s), (B) - (r), (C) - (p), (D) - (q) 12. (A) - (r), (B) - (s), (C) - (p), (D) - (q)
13. (A) - (r), (B) - (p), (C) - (s), (D) - (q)

EXERCISE-4(A)
Que. 1 2 3 4 5 6 7 8 9 10 11 12 13 14 15
Ans. 2 1 4 1 2 3 2 3 2 2 1 4 2 3 1
Que. 16 17 18 19 20 21 22 23 24 25 26 27
Ans. 2 2 3 4 1 1 2 1 3 2 1 2

EXERCISE-4(B)
SECTION - 1 : SINGLE CHOICE CORRECT QUESTIONS
Que. 1 2 3 4 5 6 7 8 9 10 11 12 13 14 15
Ans. 1 2 1 3 1 3 2 1 2 2 1 1 1 1 2
Que. 16 17
Ans. 4 4
SECTION - 2 : NUMERICAL ANSWER BASED TYPE QUESTIONS
JPR\COMP.251\D\Allen(IIT-JEE Wing)\2020–21\Nurture\Che\Unit-08\Chemical Kinetics

Que. 1 2
Ans. 23 3.984

EXERCISE-5
1. (A) 2. (A) 3. (8) 4. (4) 5. (A)
6. (0) 7. (ABD) 8. (A) 9. (8) 10. (9)
11. (D) 12. (B) 13. (9) 14. (BD) 15. (BCD)
16. (BD) 17. (8500) 18. (AD) 19. (ABC) 20. (6.75)

232

You might also like